Está en la página 1de 114

GMAT Verbal Study Guide

Table of Contents
Chapter 1 Reading Comprehension ...............................................................................4
Section 1: One Principle .............................................................................................. 5
Section 2: Two Styles .................................................................................................. 8
1. Presentation....................................................................................................... 8
2. Argumentation .................................................................................................. 9
3. Organizational Structure ..................................................................................11
Section 3: Three Subjects........................................................................................... 14
1. Natural Science ............................................................................................... 14
2. Social Science ................................................................................................. 15
3. Business Subject ............................................................................................. 17
Section 4: Four-step Process of Reading ................................................................... 20
1. Analyze the first paragraph............................................................................. 20
2. Skim the passage and get the author's main point........................................... 21
3. Diagram the organization of the passage ........................................................ 23
4. Tackle the questions and correspondently refer to the passage. ..................... 26
Section 5: Five Types of Questions ........................................................................... 29
1. Main Idea Question......................................................................................... 30
2. Recall Question............................................................................................... 35
3. Inference Questions ........................................................................................ 37
4. Critical Reasoning Question ........................................................................... 39
5. Difficult-to-locate Question ............................................................................ 41
Section 6: Six test points............................................................................................ 43
1. Comparison ..................................................................................................... 43
2. Example & Listing.......................................................................................... 43
3. People, Date & Place ...................................................................................... 46
4. Words of Attitude and Transition ................................................................... 47
5. Counter-evidence Indicators ........................................................................... 49
6. Special Punctuation......................................................................................... 51
Chapter 2 Sentence Correction ....................................................................................53
Introduction................................................................................................................ 53

1
Three-step method ..................................................................................................... 54
Section 1: Subject-Verb Agreement .......................................................................... 56
Section 2: Verb Time Sequences ............................................................................... 57
Section 3: Modification.............................................................................................. 58
A. Adjective or adverb as a modifier .................................................................. 58
B. Clause as a modifier ....................................................................................... 58
C. A long phrase as a modifier............................................................................ 59
D. Appositive as a modifier ................................................................................ 60
Section 4: Parallelism ................................................................................................ 61
Section 5: Pronoun..................................................................................................... 62
Section 6: Comparisons ............................................................................................. 63
1. Quality Comparison ........................................................................................ 63
2. Quantity Comparison ...................................................................................... 64
3. Analogy........................................................................................................... 64
Section 7: Choice of Word......................................................................................... 66
Section 8: Idioms ....................................................................................................... 67
Section 9: Sentence Structure .................................................................................... 72
Section 10: Subjunctive Mood................................................................................... 74
Section 11: Ambiguity ............................................................................................... 75
Section 12: Redundancy ............................................................................................ 76
Section 13: Awkward................................................................................................. 77
Section 14: Logicality ................................................................................................ 78
Chapter 3 Critical Reasoning .......................................................................................80
Section 1: Introduction to Critical Reasoning............................................................ 81
1. One Definition: Argument .............................................................................. 81
2. Four elements of an argument......................................................................... 82
3. Seven Common Fallacies................................................................................ 84
4. Three-element Rule......................................................................................... 86
5. Two Traps ....................................................................................................... 87
6. Five Answer Choices ...................................................................................... 88
Section 2: Six Types of Argument............................................................................. 90
1. Deductive Argument ....................................................................................... 90
2. Generalization ................................................................................................. 94
3. Analogy........................................................................................................... 95
4. Causal Reasoning............................................................................................ 96
5. Finding Assumption........................................................................................ 97
6. Business Thinking........................................................................................... 99
Section 3: Eight Types of Question ......................................................................... 100
1. Inference Question ........................................................................................ 100
2. Assumption Question.................................................................................... 102
3. Strengthen Question...................................................................................... 104

2
4. Weaken Question .......................................................................................... 106
5. Paradox Question .......................................................................................... 108
6. Reasoning Question ...................................................................................... 109
7. Complete Question.........................................................................................110
8. Boldface Question..........................................................................................112

If you have any question or suggestion, please email us at contact@microedu.com.


Copyright 2004 The Microedu.com. All Rights Reserved. Not for distribution.

3
Chapter 1 Reading Comprehension

Reading Comprehension on the test day

On the test day, you will expect to see three or four reading passages, each followed with three or
four questions. The passages presented depend on how well you are performing on the test.
However, the questions presented for the same passage do not depend on your performance. In
other word, after you are assigned a reading passage, the next question presented for the same
passage will not base on your performance on the previous question.

Why Reading Comprehension is a nightmare to most students?

Most people find the reading compression difficult to prepare because the subject matter is
unfamiliar and could be anything. In order to make sure that nobody can take advantages on a
particular subject, the test-maker takes every effort to diversify the subjects of the three or four
passages on your test day. As a result, obscure subject matter is chosen so that you will be tested,
not on your knowledge of a particular subject, but the test-taking skills.

In addition, the reading passage is not created like the one we see on magazine, newspaper, or
textbook. Rather, it uses a highly compressed style. Subjects of the passages are generally
excerpted from academic articles that were published tens of years ago. Usually the chosen
article is heavily edited until it is cut down to about 300 words, about one-third its original length.

Though it is difficult to read, the reading techniques introduced in the following passage will help
you pick up the right answer even without understanding the reading passage.

Chapter Preview

In order to make it easier for you to prepare for GMAT, we have developed an interesting course
for Reading Comprehension. You will find this chapter all in number, as the section number
suggests. We hope this would help you learn the test prep strategies.

Section 1: One Principle

Section 2: Two Writing Styles

Section 3: Three Subjects

Section 4: Four-step Procedure for Attacking a Passage

Section 5: Five Types of Question

Section 6: Six Test Points

4
Section 1: One Principle

Directions: The questions in this group are based on the content of a passage. After reading the
passage, choose the best answer to each question. Answer all questions following the
passage on the basis of what is stated or implied in the passage

On the test day, you will see the above direction on computer screen. Most students disregard
this instruction since it appears in every test. However, it introduces a basic principle you should
follow in answering a reading comprehension question. When answering questions, you must
refer each of them to some place in the passage. Don't rely on memory, since too many traps are
used with these questions.

Also, don't base on your daily life experiences or college knowledge. Remember, GMAT doesn't
test any specific knowledge on business or other functions. Image if a question is based on some
specific knowledge, then those with broad knowledge can take advantages. This definitely
violates the rule of GMAT. The test-maker often fools you by creating stuff choices that contain
reasonable statement based on basic knowledge or your life experience, not on the passage. If
you find an answer choice contains the widely known reasoning or statement on the test day,
eliminate those choices with hesitation.

Let's look at a sample passage that discusses why the Indian software vendors perform better
than their counterparts in China.

Indian firms have achieved the highest levels of efficiency in the world software outsourcing
industry. Some researchers have assumed that Indian firms use the same programming
languages and techniques as Chinese firms but have benefited from their familiarity with
English, the language used to write software code. However, if this were true, then one
would expect software vendors in Hong Kong, where most people speak English, to perform
not worse than do Indian vendors. However, this is obviously not the case.

Other researchers link high Indian productivity to higher levels of human resource investment
per engineer. But a historical perspective leads to a different conclusion. When the two top
Indian vendors matched and then doubled Chinese productivity levels in the mid-eighties,
human resource investment per employee was comparable to that of Chinese vendors.
Furthermore, by the late eighties, the amount of fixed assets required to develop one
software package was roughly equivalent in India and in the China. Since human resource
investment was not higher in India, it had to be other factors that led to higher productivity.

A more fruitful explanation may lie with Indian strategic approach in outsourcing. Indian
software vendors did not simply seek outsourced contract more effectively: they made
aggressive strategic in outsourcing. For instance, most software firms of India were initially
set up to outsource the contract in western countries, such as United States. By contrary,

5
most Chinese firms seem to position their business in China, a promising yet
under-developed market. However, rampant piracy in China took almost 90 percents of
potential market, making it impossible for most Chinese firms to obtain sufficient
compensation for the investment on development and research, let alone thrive in
competitive environment.

Now, let's look at a sample question:

Which of the following statements concerning the productivity levels of engineers can be
inferred from the passage?

(A) Prior to the 1980s, the productivity levels of the top Indian software firms were exceeded
by those of Chinese software firms.

(B) The official language of a country has a large effect on the productivity levels of its software
developers.

(C) During the late 1980s and early 1990s, productivity levels were comparable in China and
India.

(D) The greater the number of engineers that a software firm has, the higher a firms
productivity level.

(E) The amount of human resource investment made by software developers in their firms
determines the level of productivity.

If you do not refer to the original passage, you may pick up B. For test-takers who have some
backgrounds in computer, it is obvious that being familiar with English will gain some advantage
in writing program code. However, the correct answer is C.

In conclusion, the directions can run out of your eyes on the test day, but should be rooted deeply
in your heart at the beginning of your test preparation.

Trap: Some choices just repeat the same words or phrases that you read in the passage. Keep
alarm to these choices since in most cases, they are incorrect.

Here is an example:

The fact that reducing price can generate a competitive advantage for a company does not
mean that every reduction in price will create such an advantage. Price reduction, like
improvement in service, must be balanced against other types of efforts on the basis of direct,
tangible benefits such as increased revenues. If a company is already effectively on a par
with its competitors because it provides product at an acceptable price and keeps customers

6
from leaving at an unacceptable rate, then reduction in price may not be effective, since price
is not necessarily the deciding factor for any customer in any situation.

This truth was not apparent to managers of one operating system software vendor, which
failed to improve its competitive position despite its attempt to reduce price. The software
managers did not recognize the level of customer inertia that arises from the inconvenience
of switching operating system. Nor did they analyze their reduction in price to determine
whether it would attract new customers by producing a new standard of price that would
excite customers or by proving difficult for competitors to copy.

Sample question

According to the passage, reduction in price are comparable to improvement in service in


terms of the

(A) tangibility of the benefits that they tend to confer

(B) increased revenues that they ultimately produce

(C) basis on which they need to be weighed

(D) insufficient analysis that managers devote to them

(E) degree of competitive advantage that they are likely to provide

To answer this question, first locate the question to the second sentence of the passage. "Price
reduction, like improvement in service, must be balanced against other types of efforts on the
basis of direct, tangible benefits such as increased revenues." Now, go back to answer choices.
Choice D and E are irrelevant to the original sentence, so eliminate them. Then, look at the choice
A and B, both of them repeat the original sentences.

(A) tangibility of the benefits that they tend to confer

(B) increased revenues that they ultimately produce

(C) basis on which they need to be weighed

Both A and B seem to be correct. However, reduction in price is comparable to that of


improvement in service in term of the basis on direct and tangible benefits, not on the tangibility or
specific benefits of increased revenues. So neither A nor B is correct. Choice B does not repeat
the same words, but address the basis for comparison. Therefore, C is the correct answer.

7
Section 2: Two Styles

There is an endless number of writing techniques that authors use to present their ideas.
However, there are only two writing styles used in a GMAT reading passage: presentation and
argumentation.

1. Presentation

This technique is to present an idea that the author will agree or at least partially agree. The
author strengthens his position by citing relevant evidences, each related to other in a highly
structured manner. We call this style of writing as presentation. Sometimes, the author sometime
may intentionally contrast his position with an opposing view. But most often the author is just
anticipating an objection, he will soon refute it.

Here is a sample passage in presentation.

China as a nation faces two major financial problems. First, eighty-four percent of
state-owned enterprises do not generate profit. Government failed to collect money from
such business. Rather, it has to appropriate substantial funds to these enterprises in order to
prevent them from going bankrupt and thus resulting in high unemployment rate. Second,
203 million of civilians in countryside will not be able to gain pension after they retire due to
the limited budget of government.

I would like to make an outrageous suggestion that would at one stroke generate finance
earnings and provide funds for civilians retirement. I would propose that government sells its
holdings in state-owned enterprises on the open market. Such sales would provide
substantial funds for village civilians pension. At the same, they could cut down financial
burden on these state-owned enterprises.

You might object that government would be deprived of the opportunity to share its
enterprises profit if someday they make money. I agree. Sell holdings of enterprises that
would never generate profit. But, you might reply, every enterprise that competes on the
market has potential. Here we part company. Theoretically, you may be correct in claiming
that every enterprise has the potential to make money. Practically, you are wrong.

I refer to the thousands of state-owned enterprises that are not likely to make money. These
companies are 100 percent held by the nation as a whole. Government officials are
appointed as the chairman, CEO and president. The management was not responsible for
the public interest, but for the nation as a whole. If there is no significant loss in business,
they will soon be promoted back to the higher level position in government. If their
companies perform great, these executives receive direct money compensation. However,

8
their salary, when combined with such compensation, will be far below that of their
counterpart in private company.

It would be unrealistic to suggest that village civilians would have sufficient funds if
governments shares were sold on the open market. But the demand for compensating the
state-own enterprises would be substantially reduced.

The author developed the above passage by first pointing out a problem, suggesting a solution,
anticipating counter-position, illustrating an example, refuting a second solution, and further
anticipating possible objections. Obviously, this writing technique is presentation.

2. Argumentation

The second writing style is argumentation. This technique has a number of variations, but the
most common and direct is to develop two to three ideas and then point out why one is better than
the other or just simply refute all of them and developed the author's own idea.

Some common tip-off sentences to this method of analysis are:

z It was traditionally assumed...

z It was once believed...

z It was frequently assumed ..

z It was universally accepted..

z Many scientists have argued...

The passage that discusses Indian and Chinese software firms represents a typical
argumentation. At the beginning, the author presented a phenomenon and gave an explanation,
but refuted that explanation immediately.

Indian firms have achieved the highest levels of efficiency in the world software outsourcing
industry. Some researchers have assumed that Indian firms use the same programming
languages and techniques as Chinese firms but have benefited from their familiarity with
English, the language used to write software code. However, if this were true, then one
would expect software vendors in Hong Kong, where most people speak English, to perform
not worse than do Indian vendors. However, this is obviously not the case.

Then, the second explanation was introduced, but was denied again in the same paragraph.

Other researchers link high Indian productivity to higher levels of human resource investment
per engineer. But a historical perspective leads to a different conclusion. When the two top
Indian vendors matched and then doubled Chinese productivity levels in the mid-eighties,
human resource investment per employee was comparable to that of Chinese vendors.

9
Furthermore, by the late eighties, the amount of fixed assets required to develop one
software package was roughly equivalent in India and in the China. Since human resource
investment was not higher in India, it had to be other factors that led to higher productivity.

Finally, a more fruitful one is presented. The author used the remaining passage try to argue that
this explanation is the correct one.

A more fruitful explanation may lie with Indian strategic approach in outsourcing. Indian
software vendors did not simply seek outsourced contract more effectively: they made
aggressive strategic in outsourcing. For instance, most software firms of India were initially
set up to outsource the contract in western countries, such as United States. By contrary,
most Chinese firms seem to position their business in China, a promising yet
under-developed market. However, rampant piracy in China took almost 90 percents of
potential market, making it impossible for most Chinese firms to obtain sufficient
compensation for the investment on development and research, let alone thrive in
competitive environment.

Why bother to identify the writing style?

Be familiar with the author's writing techniques can help you diagram the mental road map of a
passage, identify the author's intention to cite an evidence, main idea of a passage, and most
importantly, pick up the right choice quickly and decisively. Lets go back the passage that talks
about whether price reduction can generate a competitive advantage.

The fact that reducing price can generate a competitive advantage for a company does not
mean that every reduction in price will create such an advantage. Price reduction, like
improvement in service, must be balanced against other types of efforts on the basis of direct,
tangible benefits such as increased revenues. If a company is already effectively on a par
with its competitors because it provides product at an acceptable price and keeps customers
from leaving at an unacceptable rate, then reduction in price may not be effective, since price
is not necessarily the deciding factor for any customer in any situation.

This truth was not apparent to managers of one operating system software vendor, which
failed to improve its competitive position despite its attempt to reduce price. The software
managers did not recognize the level of customer inertia that arises from the inconvenience
of switching operating system. Nor did they analyze their reduction in price to determine
whether it would attract new customers by producing a new standard of price that would
excite customers or by proving difficult for competitors to copy.

In the above passage, the author did not try to present his own position (presentation). If any, the
position is that he does not agree with the fact that reduction in price can generate competitive

10
advantage for a company. In fact, the speaker here argued against a popular point of view by
reasoning and examples (argumentation).

Let's look at a sample question to see how to pick up a right choice on the basis of writing styles.

The primary purpose of the passage is to

(A) contrast possible outcomes of a type of business strategy

(B) suggest more careful evaluation of a type of business strategy

(C) illustrate various ways in which a type of business strategy could fail to enhance revenues

(D) trace the general problems of a company to a certain type of business strategy

(E) criticize the way in which managers tend to analyze the costs and benefits of business
strategies

This question asks you to summarize the passage's central idea. Which of the five choices is
correct? Based on the verbs initiating the five choices, you can eliminate three of them:

(A) incorrect. To contrast is to compare several things, but not to agree or disagree.

(C) incorrect. To illustrate is to give example, not to agree or disagree.

(D) incorrect. To trace is to track, not to agree or disagree.

Choice E began with argumental word criticize, but isn't the correct choice because it addresses
the detail. Therefore, B is the right answer: to argue that superior service does not generate
competitive advantage is to suggest more careful evaluation of a type of business strategy (price
reduction).

3. Organizational Structure

There are two major patterns that the test-maker uses to reach a conclusion: general-to-specific
and specific-to-general. Become familiar with these writing patterns can help you identify the main
idea of a passage.

A. General-to-Specific Structure

This structure is widely used in GMAT reading passage. The test-writer first makes a general
argument, and then supports it using a series of specific examples or reasoning, and finally
summaries by reclaiming his general argument.

Here is the structure:

z General claim, followed by

z first evidence or reasoning

z second evidence or reasoning

11
z more evidence or reasoning

Let's look at a passage of this structure:

The fact that reducing price can generate a competitive advantage for a company does not
mean that every reduction in price will create such an advantage. Price reduction, like
improvement in service, must be balanced against other types of efforts on the basis of direct,
tangible benefits such as increased revenues. If a company is already effectively on a par
with its competitors because it provides product at an acceptable price and keeps customers
from leaving at an unacceptable rate, then reduction in price may not be effective, since price
is not necessarily the deciding factor for any customer in any situation.

This truth was not apparent to managers of one operating system software vendor, which
failed to improve its competitive position despite its attempt to reduce price. The software
managers did not recognize the level of customer inertia that arises from the inconvenience
of switching operating system. Nor did they analyze their reduction in price to determine
whether it would attract new customers by producing a new standard of price that would
excite customers or by proving difficult for competitors to copy.

Here, the author presents his opinion at the beginning of the passage: reduction in price does not
necessarily generate a competitive advantage. To support his idea, the author first made
reasoning by comparing service improvement and price reduction. Then, in the second paragraph,
the author used an example within operating system software industry to further address that
reducing price did not improve competitive position.

B. Specific-to-General Structure

Contrast to the general-to-specific structure, the specific-to-general first presents a group of


examples or reasoning and finally draw a conclusion.

Here is the structure:

z first example or reasoning

z second example or reasoning

z more example or reasoning

z Conclusion

The passage that discusses Indian software vendors was written in argumentation, and
represents a typical passage in specific-to-general structure.

Indian firms have achieved the highest levels of efficiency in the world software outsourcing
industry. Some researchers have assumed that Indian firms use the same programming
languages and techniques as Chinese firms but have benefited from their familiarity with

12
English, the language used to write software code. However, if this were true, then one
would expect software vendors in Hong Kong, where most people speak English, to perform
not worse than do Indian vendors. However, this is obviously not the case.

Other researchers link high Indian productivity to higher levels of human resource investment
per engineer. But a historical perspective leads to a different conclusion. When the two top
Indian vendors matched and then doubled Chinese productivity levels in the mid-eighties,
human resource investment per employee was comparable to that of Chinese vendors.
Furthermore, by the late eighties, the amount of fixed assets required to develop one
software package was roughly equivalent in India and in the China. Since human resource
investment was not higher in India, it had to be other factors that led to higher productivity.

A more fruitful explanation may lie with Indian strategic approach in outsourcing. Indian
software vendors did not simply seek outsourced contract more effectively: they made
aggressive strategic in outsourcing. For instance, most software firms of India were initially
set up to outsource the contract in western countries, such as United States. By contrary,
most Chinese firms seem to position their business in China, a promising yet
under-developed market. However, rampant piracy in China took almost 90 percents of
potential market, making it impossible for most Chinese firms to obtain sufficient
compensation for the investment on development and research, let alone thrive in
competitive environment.

In the above passage, the author gave an explanation to a particular event, but refuted it soon,
until it came with a convincing one the conclusion.

13
Section 3: Three Subjects

Like writing techniques, GMAT subjects may vary significantly. The author may present how
caffeine activates human behavior, discuss what causes Japanese auto companies to perform
better than those in USA, or explain the union's effort to organize the employees in public sectors.
Various as the subjects may be, there are only three major subjects that a GMAT passage may
be discussing about: natural science, social science, and business. As the name indicates,
natural science topic includes biology, chemistry, geology, and archeology; social science
includes art, literature, and civil rights; business includes marketing, advertising, management,
and economics.

1. Natural Science

Characteristics

Most test-takers find the natural science difficult to read and beyond their knowledge base. If you
try to figure out what they are really talking about, in most cases you will fail, because reading
passages in this kind of subject are filled of nomenclatures and jargons. The good news, however,
is that the sentences are always written in a simple syntax and the questions to be answered after
reading passage are typical of the recalled questions. That means, when you successfully locate
the "key words", you will find it easy to get the right answer.

Strategy

Don't memorize the details or try to figure out the author's reasoning. Skim the passage, and get
its central idea as well as organizational structure.

The following passage is about natural science topic. This passage is a little difficult to
understand, but the following questions are much easier to answer.

Sample Passage

The cutting-edge science is ringing alarm bells. Avian flu virus picked up by pigs can swap
genetic materials with another flu virus already in the pig and become a new, hitherto
unknown flu virus for which no person, no animal has preexisting immunity. The kind of virus
causes a pandemic because it spreads from human to human.

If you took a peek into history, it turns out that previous influenza pandemics have similar
scenarios. The greatest influenza pandemic in 1918 caused more than 20 million deaths of
soldiers stationed in France. The last influenza pandemic was in 1968, known as the Hong
Kong flu (H3N2). Thousands of deaths and millions were infected worldwide.

The other examples are the Nipah virus and Japanese Encephalitis virus, which find pigs to
be good hosts. With JE, the virus circulates in the blood of infected pigs. When infected pigs

14
are bitten by Culex mosquitoes, the virus replicates in the mosquito's gut. The next time the
mosquito bites a human, the virus is passed on. The pig doesn't get sick as such. The Nipah
virus causes pneumonia symptoms in pigs. In humans, it causes encephalitis, and humans
catch it only with direct contact with infected pigs. Symptoms range from mild headache to
permanent brain damage, and can be fatal.

It's merely a phenomenon of nature that the pig is the "mixing vessel" for the new germ. But
make no mistake, the pig is not the villain, neither is the chicken. It's actually us, and our
horrible farm practices, outdated agricultural policy and, most of all, reckless disregard of our
ecology and environment. "Hygiene and management can control what eventually happens,"
says Lam. "Good farming practice will prevent serious outbreaks and infection to humans."
Despite knowing that, animal diseases and the possibility of transmission to humans are
becoming quite alarming. Of the 35 new emerging diseases in the last 20 years, more than
70 per cent involved animals.

In fact, what we may have done is unwittingly create the perfect launch pad for an influenza
pandemic that will likely kill large numbers of people across the globe. Although scientists
say it's impossible to predict the odds that the virus will alter its genetic form radically enough
to start leaping from human to human, the longer H5N1 is out there killing chickens, the
higher the chances are.

Sample Question

Which of the following statement can be inferred from the passage?

(A) New emerging diseases causes more deaths of human than animal.

(B) Animals are the villain for most flues.

(C) Hygiene and management can not control the spread of viruses.

(D) The current bird flu epidemic may be a launch pad for the next influenza pandemic.

(E) The influenza pandemic is always a regional phenomenon.

Which answer is correct? For choice A, the passage did not make any comparison between
deaths of human and deaths of animal. In B, animal is actually not the villain for most flues. Rather,
it is human. Look at the second sentence in the fourth paragraph, But make no mistake, the pig
is not the villain, neither is the chicken. For C, Hygiene and management can control what
eventually happens(in the middle of fourth paragraph), therefore, C is incorrect. E is also
incorrect. Though most flues discussed in this passage were originated from some areas, the
passage never stated it was a regional phenomenon. In fact, it will likely kill large numbers of
people across the globe, as stated at the beginning of last paragraph. The correct answer is D
the current bird flu epidemic may be a launch pad for the next influenza pandemic, because no
animal has preexisting immunity and it causes a pandemic by spreading from human to human.

15
2. Social Science

Characteristics

Why women's rights experienced a significant improvement during 1860's? How the Pullman
stroke to improve their living condition? Passages in these subjects are easy to read because it
goes as you expect and talks about something around your world. You will find it easy to grasp
the main idea and passage map. In order to get the right answer, however, you need to read
beyond the words, phrases or concepts in the passage. The right answer is always created in a
synthesized way.

Strategy

Be careful in tackling this "social" passage. To answer the later questions is always not as easy
as to understand the passage. The answer choice that contains the exact words or phrases from
the passage is generally not the correct answer. Rather, you need to synthesize several
sentences or make some reasoning before you pick up the right choice. The process is
time-consuming because the social passage is typically long.

Sample Passage

China as a nation faces two major financial problems. First, eighty-four percent of
state-owned enterprises do not generate profit. Government failed to make money from such
business. Rather, it has to appropriate substantial funds to these enterprises in order to
prevent them from going bankrupt and thus resulting in high unemployment rate. Second,
203 million of civilians in countryside will not be able to gain pension after they retire due to
the limited budget of government.

I would like to make an outrageous suggestion that would at one stroke generate finance
earnings and provide funds for civilians retirement. I would propose that government sells its
holdings in state-owned enterprises on the open market. Such sales would provide
substantial funds for village civilians pension. At the same time, they could cut down
financial burden on these state-owned enterprises.

You might object that government would be deprived of the opportunity to share its
enterprises profit if someday they make money. I agree. Sell holdings of enterprises that
would never generate profit. But, you might reply, every enterprise that competes on the
market has potential. Here we part company. Theoretically, you may be correct in claiming
that every enterprise has the potential to make money. Practically, you are wrong.

I refer to the thousands of state-owned enterprises that are not likely to make money. These
companies are 100 percent held by the nation as a whole. Government officials are
appointed as the chairman, CEO and president. The management was not responsible for

16
the public interest, but for the nation as a whole. If there is no significant loss in business,
they will soon be promoted back to the higher level position in government. If their
companies perform great, these executives receive direct money compensation. However,
their salary, when combined with such compensation, will be much less than the amount
they would earn if were in private company.

It would be unrealistic to suggest that village civilians would have sufficient funds if
governments shares were sold on the open market. But the demand for compensating the
state-own enterprises would be substantially reduced.

Sample Question

According to the passage, executives in a state-owned enterprise are motivated by

(A) direct money compensation

(B) increasing salary

(C) political outlook

(D) share option

(E) social responsibility

The passage mentioned the executives of state-owned enterprises in fourth paragraph; therefore,
we need not to consider other paragraphs when referring to the original passage. Since their
salary, when combined with such compensation, will be much less than the amount they
would earn if were in private company as stated in the last sentence, these executives are not
motivated by financial earnings. If yes, they will transfer to a private company. Therefore, they are
not motivated by direct money compensation, increasing salary, or share option. Rather, they are
concerned on their political outlook. If there is no significant loss in business, they will soon
be promoted back to the higher level position in government. Choice C is the correct answer.
For choice E, the passage never discussed the executives social responsibility.

17
3. Business Subject

Characteristics

This subject is highly welcomed since most students possess some knowledge or background in
business. But passage of this subject contains the most difficult questions in GMAT Reading
Comprehension. Recall questions are few and you always have to reason before you pick up the
correct choice.

Strategy

Don't rely on your memory even if you become or have been quite familiar with its topics. There
are too many traps here. Make sure you refer to the passage when answering the questions.

Sample Passage

The fact that reducing price can generate a competitive advantage for a company does not
mean that every reduction in price will create such an advantage. Price reduction, like
improvement in service, must be balanced against other types of efforts on the basis of direct,
tangible benefits such as increased revenues. If a company is already effectively on a par
with its competitors because it provides product at an acceptable price and keeps customers
from leaving at an unacceptable rate, then reduction in price may not be effective, since price
is not necessarily the deciding factor for any customer in any situation.

This truth was not apparent to managers of one operating system software vendor, which
failed to improve its competitive position despite its attempt to reduce price. The software
managers did not recognize the level of customer inertia that arises from the inconvenience
of switching operating system. Nor did they analyze their reduction in price to determine
whether it would attract new customers by producing a new standard of price that would
excite customers or by proving difficult for competitors to copy.

Sample Question

The passage suggests which of the following about price charged by an operating system software
vendor prior to its strategy in reducing its price?

(A) It was slightly low to that of the vendors competitors.

(B) It threatened to weaken the vendors competitive position with respect to other operating
system software vendor

(C) It had already been reduced after having caused damage to the vendors reputation in the
past.

(D) It enabled the vendor to retain customers at an acceptable rate

18
(E) It needed to be reduced to attain parity with the software provided by competing vendors.

Here, the question was created in complicated clauses and itself already hard to understand. In
fact, it asks for the situation of the vendor before price reduction. Only D can be inferred from the
passage. The original passage stated that If a company is already effectively on .. keeps
customers from leaving at an unacceptable rate and This truth was not apparent to
managers of one operating system software vendor That means the vendor was able to
retain customers at an acceptable rate.

19
Section 4: Four-step Process of Reading

In the previous section we summarize three kinds of subject you will encounter in a GMAT
reading passage. Now you will learn the four-step procedure to read a passage in any subject:

1. Analyze the first paragraph.

2. Skim the passage and get some idea of the main idea

3. Identify the purpose of each paragraph and structure of the passage

4. Answer the questions and don't forget to refer to the passage

1. Analyze the first paragraph.

It is essential to carefully read the first paragraph. You will get informed what the passage is
talking about, and even the main idea of the passage. There are two major reasons for you to
carefully read the first paragraph.

Fist of all, the paragraph is the main structural unit of any passage. Every paragraph is needed to
understand the whole passage or answer the question after the passage. Test-maker never
delivers a junk content. It must talk about something that relates to the central idea, and present it
as persuasively as possible. In fact, the first paragraph introduces either the position that the
author will support or the one that he/she will argue against. So, getting familiar with the
introductory paragraph will definitely help you identify the main topic.

Secondly, analyzing the fist paragraph in stead of the whole passage can save you much time. As
I said at the beginning of this chapter, GMAT reading passage is dry and unfamiliar. It is highly
likely that after you read the passage, you get no ideas about what the passage is talking about. If
you go back and reread the whole passage, you will have no sufficient time to answer the
question. Analyze the first paragraph, pay attention to concepts, and then you will find it easy to
understand the subject of passage.

Below is the first paragraph of a GMAT reading passage. Pay attention to concept words.

China as a nation faces two major financial problems. First, eighty-four percent of
state-owned enterprises do not generate profit. Government failed to make money from such
business. Rather, it has to appropriate substantial funds to these enterprises in order to
prevent them from going bankrupt and thus resulting in high unemployment rate. Second,
203 million of civilians in countryside will not be able to gain pension after they retire due to
the limited budget of government.

The first sentence stated that China faces two problems. Then, the author specified these two
problems using a clear and logical structure. Firstly, government did not make money from but

20
input large amount of money to its enterprises. Secondly, government has limited funds for
pension.

Now, let's summarize this paragraph and put it in our own words-- China has two problems:
financial burden and limited funds. Keep these key words (concepts) in mind, and you will find it
easy to understand the remaining passage that we'll present in next step.

2. Skim the passage and get the author's main point

Here are some strategies that will speed your reading and help you identify the author's main
points:

z Focus on the first sentence of each paragraph

The first sentence of a paragraph is always the main point of this paragraph. Why? It confirms to
the formal writing style. If you are a management consultant, you will find it a great advantage to
use a summary at the very beginning of each section. Image when you are presenting a strategy
report which contains hundreds of pages, how could your clients catch your all of them? The only
solution is to use a highly structured presentation, and summarize your idea at the beginning of
each section. In fact, you are also doing like this in the AWA section.

By simply reading the first sentence of each paragraph, you can construct a mental road map of
the passage while not spending significant time.

z Pay attention to the mood words

"Mood" words are those that the author uses to demonstrate his/her position to a particular event,
phenomenon, or point of view. A mood word can be positive or negative. Positive words such as
successfully, correctly and right often illustrate an idea that the author agree. And vis-a-vis, a
negative word indicates an idea that will be weakened in later passage.

The following sentences express the author's position by using positive mood words:

a) Haney's through research provides convincing field evidence that..

b) For many yeas, Benjamin Quarles' seminal account of the participation of African Americans
in the American Revolution has remained the standard work in the field.

c) Roger Rosenblatt's book successfully alters the approach taken by most previous studies.

By contrast, the following mood words are negative.

fail ignore overestimate underestimate

misunderstand misrepresent overlook exaggerate

sound convincingly successfully correctly

21
z Never ignore the counter-evidence indicators

The author uses counter-evidence words not to argue against himself, but concede certain minor
points that may weaken his argument. The counter evidence is finally refuted by further evidence.
You should keep alarm to these words since some students often mistake them as introducing
arguing against a statement.

Following are some of the most common used counter-evidence indicators:

actually despite admittedly except

even though nonetheless nevertheless although

however In spite of do may

OK. Let's go back to the passage talking about national finance. Here are the other five
paragraphs. In order for you to skim the passage using the above three techniques, we
underlined the first sentences of each paragraph, boldfaced the mood words and italicize the
counter-evidence indicators.

I would like to make an outrageous suggestion that would at one stroke generate finance
earnings and provide funds for civilians retirement. I would propose that government sells its
holdings in state-owned enterprises on the open market. Such sales would provide
substantial funds for village civilians pension. At the same time, they could cut down
financial burden on these state-owned enterprises.

You might object that government would be deprived of the opportunity to share its
enterprises profit if someday they make money. I agree. Sell holdings of enterprises that
would never generate profit. But, you might reply, every enterprise that competes on the
market has potential. Here we part company. Theoretically, you may be correct in claiming
that every enterprise has the potential to make money. Practically, you are wrong.

I refer to the thousands of state-owned enterprises that are not likely to make money. These
companies are 100 percent held by the nation as a whole. Government officials are
appointed as the chairman, CEO and president. The management was not responsible for
the public interest, but for the nation as a whole. If there is no significant loss in business,
they will soon be promoted back to the higher level position in government. If their
companies perform great, these executives receive direct money compensation. However,
their salary, when combined with such compensation, will be much less than the amount
they would earn if were in private company.

22
It would be unrealistic to suggest that village civilians would have sufficient funds if
governments shares were sold on the open market. But the demand for compensating the
state-own enterprises would be substantially reduced.

What is the main idea of the passage? In a word, the author is to present a solution to funding
civilians pension while benefiting the state-owned enterprises.

3. Diagram the organization of the passage

You got main idea of each paragraph. Now, its time to ask yourself why the author includes them,
what the purpose of each paragraph is, and how each paragraph relates to other. This will help
you diagram the organization of a passage, and locate the details when you answer the
questions.

Pivotal words can help you in diagramming the organization. Pivotal words are signal words or
phrases that would in advance indicate the idea of paragraphs. Below represents the most
frequently used pivotal words or sentences you will see in a reading passage.

Note: A and B represent something, while sb represents somebody.

Introduction

z When it comes to ..., some think ...

z There is a public debate today that ...

z A is a common way of ..., but is it a wise one?

z Recently the problem has been brought into focus.

Presenting Opinion

z Now there is a growing awareness that...

z It is time we explore the truth of ...

z Nowhere in history has the issue been more visible.

Further Presenting Opinion

z ... but that is only part of the history.

z Another equally important aspect is ...

z A is but one of the many effects. Another is ...

z Besides, other reasons are...

Anticipating Objections

z You may reply that.

23
z Admittedly, ..

z It is reasonable to expect...

z It is not surprising that...

Exampling

z For example(instance),...

z ... such as A,B,C and so on (so forth)

z A good case in point is...

z A particular example for this is...

Presenting Reasons

z There are many reasons for ...

z Why .... , for one thing,...

z The answer to this problem involves many factors.

z Any discussion about this problem would inevitably involves ...

z The first reason can be obliviously seen.

z Most people would agree that...

z Some people may neglect that in fact ...

z Others suggest that...

z Part of the explanation is ...

Comparing

z The advantages for A for outweigh the disadvantages of...

z Although A enjoys a distinct advantage ...

z Indeed , A carries much weight than B when sth is concerned.

z A maybe ... , but it suffers from the disadvantage that...

Transitioning

z To understand the truth of ..., it is also important to see...

z A study of ... will make this point clear

Further Anticipating Objections

z Certainly, B has its own advantages, such as...

z I do not deny that A has its own merits.

24
Conclusion

z From what has been discussed above, we may safely draw the conclusion that ...

z In summary, it is wiser ...

z In short...

In step 2, you are assigned to skim the passage and get the main idea. Here, let's identify the
purpose of each paragraph for the archeology passage to better understand the passage.

(First of all, the author presented the problems)

China as a nation faces two major financial problems. First, eighty-four percent of
state-owned enterprises do not generate profit. Government failed to make money from such
business. Rather, it has to appropriate substantial funds to these enterprises in order to
prevent them from going bankrupt and thus resulting in high unemployment rate. Second,
203 million of civilians in countryside will not be able to gain pension after they retire due to
the limited budget of government.

(Then, the author suggested a solution to the problems)

I would like to make an outrageous suggestion that would at one stroke generate finance
earnings and provide funds for civilians retirement. I would propose that government sells its
holdings in state-owned enterprises on the open market. Such sales would provide
substantial funds for village civilians pension. At the same time, they could cut down
financial burden on these state-owned enterprises.

(Here, the author anticipated a possible objection)

You might object that government would be deprived of the opportunity to share its
enterprises profit if someday they make money. I agree. Sell holdings of enterprises that
would never generate profit. But, you might reply, every enterprise that competes on the
market has potential. Here we part company. Theoretically, you may be correct in claiming
that every enterprise has the potential to make money. Practically, you are wrong.

(Then, the author gave an example to deny this objection)

I refer to the thousands of state-owned enterprises that are not likely to make money. These
companies are 100 percent held by the nation as a whole. Government officials are
appointed as the chairman, CEO and president. The management was not responsible for
the public interest, but for the nation as a whole. If there is no significant loss in business,
they will soon be promoted back to the higher level position in government. If their
companies perform great, these executives receive direct money compensation. However,

25
their salary, when combined with such compensation, will be much less than the amount
they would earn if were in private company.

(Finally, the author further anticipated a possible objection)

It would be unrealistic to suggest that village civilians would have sufficient funds if
governments shares were sold on the open market. But the demand for compensating the
state-own enterprises would be substantially reduced.

Now, you are able to create a mental road map for the whole passage:

Paragraph # 1: introduced two major problems that China faces.

Paragraph # 2: suggested a solution and explained why it is effective.

Paragraph # 3: anticipated a possible objection and denied it soon.

Paragraph # 4: exemplified to argue against a position initiated in the third paragraph.

Paragraph # 5: concluded that his solution is not perfect, but really effective

By making such a road map, I bet you understand this passage quite well.

4. Tackle the questions and correspondently refer to the passage.

Now that you have grasped main idea and the organizational structure of the passage, you are
about to answer the following questions. Again, dont base on your memory. Always refer to the
original passage before you pick up a choice.

China as a nation faces two major financial problems. First, eighty-four percent of
state-owned enterprises do not generate profit. Government failed to make money from such
business. Rather, it has to appropriate substantial funds to these enterprises in order to
prevent them from going bankrupt and thus resulting in high unemployment rate. Second,
203 million of civilians in countryside will not be able to gain pension after they retire due to
the limited budget of government.

I would like to make an outrageous suggestion that would at one stroke generate finance
earnings and provide funds for civilians retirement. I would propose that government sells its
holdings in state-owned enterprises on the open market. Such sales would provide
substantial funds for village civilians pension. At the same time, they could cut down
financial burden on these state-owned enterprises.

You might object that government would be deprived of the opportunity to share its
enterprises profit if someday they make money. I agree. Sell holdings of enterprises that
would never generate profit. But, you might reply, every enterprise that competes on the

26
market has potential. Here we part company. Theoretically, you may be correct in claiming
that every enterprise has the potential to make money. Practically, you are wrong.

I refer to the thousands of state-owned enterprises that are not likely to make money. These
companies are 100 percent held by the nation as a whole. Government officials are
appointed as the chairman, CEO and president. The management was not responsible for
the public interest, but for the nation as a whole. If there is no significant loss in business,
they will soon be promoted back to the higher level position in government. If their
companies perform great, these executives receive direct money compensation. However,
their salary, when combined with such compensation, will be much less than the amount
they would earn if were in private company.

It would be unrealistic to suggest that village civilians would have sufficient funds if
governments shares were sold on the open market. But the demand for compensating the
state-own enterprises would be substantially reduced.

1. The primary purpose of the passage is to propose

(A) an alternative to manage government property

(B) a way to relieve government burden while providing funds to village civilians

(C) a way to distinguish state-owned enterprises that make money from those that do no
make money

(D) the governmental approach to evaluate state-owned enterprise executives

(E) a new system for national pension system

This question requires you to identify the primary concern of the passage as a whole. The first
paragraph introduces two major problems that China faces. The second paragraph suggests a
solution and explains why it is effective. The third anticipates a possible objection and refutes it
soon. The fourth paragraph illustrates an example to support the authors argument. In the last
paragraph, the author concludes that his solution is not perfect, but really effective. Therefore, the
correct answer is B.

2. The author implies that all of the following statements about enterprises with which government
holds 100 percent share are true EXCEPT:

(A) A market for governments share already exists.

(B) Such enterprises seldom generate profit.

(C) There is likely to be a continuing loss of such enterprises.

(D) Government officers are appointed as the executives with such enterprises.

27
(E) If the executives perform poorly, they will be demoted to lower position.

The question requires you to identify the answer choice that CANNOT be inferred from the
passage. Nothing in the passage implies that the executives will be demoted to lower position if
they perform poorly. Therefore, the best answer is E. In answering the question that contains
EXCEPT, keep alarm not to be fooled by the test maker.

3. The author implies that which of the following would occur if governments shares were sold on
the open market?

I. The shortage of retirement fund in village would eventually cease completely.

II. Current executives in state-owned enterprises are not motivated to perform better

III. Civilians in countryside would be able to seek sufficient funds from government.

(A) I only

(B) II only

(C) I and II only

(D) II and III only

(E) I, II, and III

This question asks you to identify information that is suggested rather than directly stated in the
passage. To answer it, first look for the location in the passage of the information specified in
answer choice. The last paragraph states that It would be unrealistic to suggest that village
civilians would have sufficient funds if governments shares were sold on the open market,
therefore, I is incorrect. III, which is a repeated of I, is also incorrect. Only II can be inferred from
the original passage, therefore B is the best answer.

28
Section 5: Five Types of Questions

While the techniques introduced in previous four sections speed your reading, this section is
developed to help you pick up the right choice quickly and decisively. In the following passage, we
will discuss the major question types you may encounter in real GMAT test. Generally, there are
only five major types of questions. As you become familiar with the following question types, you
will gain an intuitive sense for the places from which questions are likely to be drawn. Note, the
order in which the questions are asked roughly corresponds to the order in which the main issues
are presented in the passage. Early questions should correspond to information given early in the
passage, and so on.

Of course, there are many other kinds of classification according to different criteria. Here, we
classify, by how we solve reading comprehension questions, into five based on the summary of
thousands of the previous real questions. Let's preview the five question types.

Question Types Preview

1. Main Idea Question


a) Main Topic
b) Tone
c) Structure
d) Exemplifying
2. Recall Question
a) Description
b) Listing
3. Inference Question
4. Critical Reasoning Question
a) Analogy
b) Assumption/Weaken/Strengthen
5. Unable-to-locate Question

29
1. Main Idea Question

There are four sub-types for this kind of question: Main Topic, Tone, Structure, and Exemplifying.
Why should we incorporate them into one type of question? In answering Main Idea Question,
you should understand the organizational structure of the passage, the author tone toward a
particular point of view in the reading passage, the purpose of each paragraph and why a
particular example was illustrated. In other words, if you can determine the main topic of the
passage, you are simultaneously well informed with the structure, the intent of specific example,
and tone toward specific position.

A. Main Topic

Main idea questions test your ability to identify and understand an author's intent in a passage.
The main idea is usually stated in the first or last paragraph. Main idea questions are usually the
first questions asked. Some common main idea questions include:

z The primary purpose of the passage is to

z Which of the following titles would best describe the content of the passage?

z The passage supplies information that would answer which of the following questions?

z Which of the following is the principal topic of the passage?

z The passage is most probably an excerpt from.

z Which of the following best states the central idea of the passage?

In most cases, main idea questions are easy to solve. In most GMAT passage the author's
primary purpose is to persuade the reader to accept her opinion. Occasionally, it is to describe
something. By determining the relationship of each paragraph, you come up with the main ideal at
the same time. However, the GMAT writers may obscure the correct answer by surrounding it
with close answer choices that stress specifics. Eliminate these choices without hesitation on the
test day.

Trap 1: The main topic will not focus on certain details in the passage. If you encounter the main
ideal question, eliminate the answer choices that describe the details.

Trap 2: Pay special attention to the "repeat" answer. Certain choices may exactly repeat some or
most words of the correct answer, but do not present the central idea, therefore, is not the correct
answer.

B. Tone Question

Tone questions ask you to identify the writer's attitude. Is the writer's feeling toward the subject
positive, negative, or neutral? The following represents some ways of the questions asked.

30
z Which of the following best summarizes the author's evaluation of Bailyn's fourth
proposition?

z The author's attitude toward the culture in most factories is best described as

(A) cautious (B) critical (C) disinterested (D) respectful (E) adulatory

However, if you did not get a feel for the writer's attitude on the first reading, check the mood
words that he chooses. Beware of answer choices that contain extreme emotions. Remember the
passages are taken from academic journals. In the rarefied air of academic circles, strong
emotions are considered inappropriate and sophomoric. The writers want to display opinions that
are considered and reasonable, not spontaneous and off-the-wall. So if an author's tone is
negative, it may be disapproving, not snide or ridiculous. If her tone is positive, it may be
approving, not ecstatic. Or if her tone is neutral, it would be not be disinterested.

C. Organizational Structure

When you can determine the right answer for main topic, generally you have been familiar with
the organizational structure. Every passage is consisted of some paragraphs, and each single
paragraph performs some certain function to the passage as a whole, by presenting, supporting
or refuting the central idea. So, think about the purpose of each paragraph as you read through
the passage.

You may encounter one type of question concerning the main idea or purpose of some certain
paragraph. Some common questions include:

z The last paragraph of the passage performs which of the following functions?

z Which of the following best describes the organization of the second paragraph?

z Which of the following best describes the organization of the passage?

z Which of the following best describes the relation of the first paragraph to the passage as
a whole?

It is relatively easy to solve this kind of question for two reasons. First, however the question may
ask, it is concerning the main idea of the paragraph. If you come up with a question including
"paragraph", it definitely require you to generate that paragraph. Second, paragraph is only some
element of the completed passage. If you can generate the main topic for the whole passage of
three to four paragraphs, why aren't you able to summarize just one paragraph?

D. Exemplifying

The other type of structure question, exemplifying, tests your ability to identify the intention of
author's illustrating of something, some people, or phenomenon. In answering this question, you
need to first locate the example, and then refer to opinion preceding or accompanying the
example. The right answer is the repeat of this opinion.

z In illustrating the example in line 13-16, the author intended to.?

31
z The author referred to the experiment in order to.?

In addition to the above four types, the Main Idea Question may require you to respond in other
ways, such as:

z From what kinds of subject could this passage be excerpted?

z Which of the following topic would be preceding this passage?

This question, however, is similar to the above four, since they are based on your understanding
of the completed passage.

Sample Question #1

The cutting-edge science is ringing alarm bells. Avian flu virus picked up by pigs can swap
genetic materials with another flu virus already in the pig and become a new, hitherto
unknown flu virus for which no person, no animal has preexisting immunity. The kind of virus
causes a pandemic because it spreads from human to human.

If you took a peek into history, it turns out that previous influenza pandemics have similar
scenarios. The greatest influenza pandemic in 1918 caused more than 20 million deaths of
soldiers stationed in France. The last influenza pandemic was in 1968, known as the Hong
Kong flu (H3N2). Thousands of deaths and millions were infected worldwide.

The other examples are the Nipah virus and Japanese Encephalitis virus, which find pigs to
be good hosts. With JE, the virus circulates in the blood of infected pigs. When infected pigs
are bitten by Culex mosquitoes, the virus replicates in the mosquito's gut. The next time the
mosquito bites a human, the virus is passed on. The pig doesn't get sick as such. The Nipah
virus causes pneumonia symptoms in pigs. In humans, it causes encephalitis, and humans
catch it only with direct contact with infected pigs. Symptoms range from mild headache to
permanent brain damage, and can be fatal.

It's merely a phenomenon of nature that the pig is the "mixing vessel" for the new germ. But
make no mistake, the pig is not the villain, neither is the chicken. It's actually us, and our
horrible farm practices, outdated agricultural policy and, most of all, reckless disregard of our
ecology and environment. "Hygiene and management can control what eventually happens,"
says Lam. "Good farming practice will prevent serious outbreaks and infection to humans."
Despite knowing that, animal diseases and the possibility of transmission to humans are
becoming quite alarming. Of the 35 new emerging diseases in the last 20 years, more than
70 per cent involved animals.

In fact, what we may have done is unwittingly create the perfect launch pad for an influenza
pandemic that will likely kill large numbers of people across the globe. Although scientists
say it's impossible to predict the odds that the virus will alter its genetic form radically enough

32
to start leaping from human to human, the longer H5N1 is out there killing chickens, the
higher the chances are.

Which of the following best describes the topic of the passage?

(A) What causes the Nipah virus and Japanese Encephalitis virus to happen?

(B) Does Hong Kong flu originate from pig?

(C) From fowl to pigs to humans?

(D) Is influenza pandemic horrible?

(E) Shall we eat chicken?

This question asks you to find a title for the passage. In other word, it requires you to identify the
primary concern of the passage as a whole. The first paragraph presents a recent virus. The
second and third paragraphs describe similar influenza pandemics in history. The fourth
paragraph concludes who should be responsible for the spread of virus and what human can do
to control. The last paragraph indicates that people stimulated rather than inhibited its
promulgation. We can thus conclude the current virus will also leap to human. Furthermore, the
passage as a whole is to ring alarm bells. Therefore, C is the best answer.

Sample Question #2

Indian firms have achieved the highest levels of efficiency in the world software outsourcing
industry. Some researchers have assumed that Indian firms use the same programming
languages and techniques as Chinese firms but have benefited from their familiarity with
English, the language used to write software code. However, if this were true, then one
would expect software vendors in Hong Kong, where most people speak English, to perform
not worse than do Indian vendors. However, this is obviously not the case.

Other researchers link high Indian productivity to higher levels of human resource investment
per engineer. But a historical perspective leads to a different conclusion. When the two top
Indian vendors matched and then doubled Chinese productivity levels in the mid-eighties,
human resource investment per employee was comparable to that of Chinese vendors.
Furthermore, by the late eighties, the amount of fixed assets required to develop one
software package was roughly equivalent in India and in the China. Since human resource
investment was not higher in India, it had to be other factors that led to higher productivity.

A more fruitful explanation may lie with Indian strategic approach in outsourcing. Indian
software vendors did not simply seek outsourced contract more effectively: they made
aggressive strategic in outsourcing. For instance, most software firms of India were initially
set up to outsource the contract in western countries, such as United States. By contrary,
most Chinese firms seem to position their business in China, a promising yet

33
under-developed market. However, rampant piracy in China took almost 90 percents of
potential market, making it impossible for most Chinese firms to obtain sufficient
compensation for the investment on development and research, let alone thrive in
competitive environment.

Which of the following best describes the organization of the first paragraph?

(A) A thesis is presented and supporting examples are provided.

(B) Opposing views are presented, classified, and then reconciled.

(C) A fact is stated, and an explanation is advanced and then refuted.

(D) A theory is proposed, considered, and then amended.

(E) An opinion is presented, qualified, and then reaffirmed.

This question requires you to identify the organizational structure of the first paragraph. In this
paragraph, the author first states a fact that Indian firms achieved the highest efficiency in software
outsourcing. Then, an assumption is presented to explain such phenomenon. However, the author
refuted this explanation soon. Thus, C is the best answer.

34
2. Recall Question

There are two subtypes of recall questions: detail-locating and listing. In the following passage,
we'll discuss one by one.

A. Detail-locating

Locating question is the most common question you will encounter in Reading Comprehension. It
roughly constitutes to 50-60% of total numbers of questions. That means, in every reading
passage, there will be about one or two detail-locating questions. It is quite simple, however, to
solve this seemingly difficult question if you are able locate the detail tested. The right answer
choice is rewritten from certain sentence in the passage by changing some words or phrases. For
example, test writer will change some words from adjective to adverbial, from noun to gerund, or
just change to its synonym.

Strategy: How to locate

Below we will introduce the three-step method to locate detail.

(1) Before you locate the question to passage, you need to determine what to locate. Key words
are something that is mentioned both in the question and in passage. Then, what are key words?
Look at the following question:

Which of the following is mentioned in the passage as a disadvantage of storing artifacts


in museum basements?

Here, key words will not be any word or phrase of "which of the following is mentioned in the
passage as", but will be "disadvantage" from "disadvantage of storing artifacts in museum
basements".

We call this step as Defining Key Words.

(2) After you define key words, you are turning to the original passage. Sometimes, the key words
will appear several times in different parts of the passage. Where should you refer to? Generally,
you should locate the key words to the sentence in which key words first appear. After all, you
have only several minutes to complete a passage.

(3) When you determined which sentence (sometimes, two or more sentences) to locate, get
some idea, then quickly refer to the answer choices. Do not spend too much time analyzing this
sentence since it may be too long or complicated to understand. If the choice mentions something
that only appears in other part of the passage, eliminate it. Also eliminate the choice that just
repeats the words or phrases from original passage.

B. Listing

The other type of Recall Question is listing. As the name indicates, Listing Question requires you
to identify some people, actions, or situations that are enumerated in the passage.

35
Here are some Listing Questions:

z According to the passage, senior managers use intuition in all of the following ways
EXCEPT to:

z According to the passage, critics of the Ewha women's studies program cited the program
as a threat to which of the following?

I. National identity

II. National unification

III. Economic development

IV. Family integrity

(A) I only (B) I and II only (C) I, II, and III only

(D) II, III, and IV only (E) I, II, III, and IV

To solve this type of question, you should first name the key word from the stimulus, and locate it
to original passage. Then, you will find some lists that are similar to the answer choices. Carefully
compare those lists one by one to the answer choices. Use POE to eliminate incorrect choice,
until you find the right one.

36
3. Inference Questions

Inference question is the second most common. Unlike recall question, inference questions
require you to go beyond the passage. That means, the correct answer must say more than what
is said in the passage. Beware of same language traps with these questions: the correct answer
will often both paraphrase and extend a statement in the passage, but it will not directly quote it. If
you are puzzled how to determine whether a detail question is recall question or inference
question, pay attentions to the way the question asks. Generally, inference question will include
some word, such as infer, suggest and imply that indicates what kind of question it is.

z It can be inferred from the passage that...

z The passage/author suggests that.

z The passage/author implies that...

Since we must not directly refer to the original passage in answering inference question, we need
to decipher the inference. Next, we will show you how to reason from couples of sentence.

Technique 1 Reasoning by Word of Comparison

The question is asking about B, but you may be unable to directly identify the characters of B
even you have located B. Rather, the original sentence is discussing about A. Here, you should
turn to the word that indicates comparison between A and B. Some words that indicate strong
comparison are unlike, in contrast to, by contrast and compared with. When you can determine
the character of B, you can simultaneously determine A is B or non-B. Also, the passage may
compare two particular events by dates or places. The phrases could be "prior to 1975" or "since
mid-1970's".

Technique 2 Reasoning by Syllogism

In logics, Syllogism looks like this: every virtue is laudable; kindness is a virtue; therefore,
kindness is laudable. As we put it in more simple way, it may be "AB and, then AC". It may be
relatively easy to recognize AB by locating the key word in the question, but it will always take
some time to identify BC, since they may be located in other part of the place. So pay attention
to the pronouns (it or they) and the nouns with definite article "the" since they often serve as B.

The fact that reducing price can generate a competitive advantage for a company does not
mean that every reduction in price will create such an advantage. Price reduction, like
improvement in service, must be balanced against other types of efforts on the basis of direct,
tangible benefits such as increased revenues. If a company is already effectively on a par
with its competitors because it provides product at an acceptable price and keeps customers
from leaving at an unacceptable rate, then reduction in price may not be effective, since price
is not necessarily the deciding factor for any customer in any situation.

37
This truth was not apparent to managers of one operating system software vendor, which
failed to improve its competitive position despite its attempt to reduce price. The software
managers did not recognize the level of customer inertia that arises from the inconvenience
of switching operating system. Nor did they analyze their reduction in price to determine
whether it would attract new customers by producing a new standard of price that would
excite customers or by proving difficult for competitors to copy.

The passage suggests which of the following about price charged by an operating system software
vendor prior to its strategy in reducing its price?

(A) It enabled the vendor to retain customers at an acceptable rate

(B) It threatened to weaken the vendors competitive position with respect to other operating
system software vendor

(C) It had already been reduced after having caused damage to the vendors reputation in the
past.

(D) It was slightly low to that of the vendors competitors.

(E) It needed to be reduced to attain parity with the software provided by competing vendors.

Here, the question asks for the situation of the vendor before price reduction. Only A can be
inferred from the passage. The original passage stated that If a company is already effectively
on .. keeps customers from leaving at an unacceptable rate and This truth was not
apparent to managers of one operating system software vendor That means the vendor was
able to retain customers at an acceptable rate.

38
4. Critical Reasoning Question

Even in reading comprehension, you will encounter some critical reasoning questions: analogy,
assumption, weaken, and strengthen. Here, the whole passage is an argument with premises,
assumptions and conclusions. The question asks you to identify the reasoning, critique the
argument or recognize the potential assumption. When you need to do is also to first locate the
conclusion to particular sentence of the passage, then identify the evidence and conclusion. The
premise (or evidence) could be near to the conclusion, or in other part of the 3-4 paragraph
passage. So, it is more difficult than its counterpart in critical reasoning section. That's why it
appears more often in high difficult level screen.

A. Analogy

Also known as application question, analogy question requires you to identify the author's
reasoning somewhere in the passage and then ask you to select one from the following five
answer choices that reasons as that is presented in the passage.

The answer choices are generally long and complicated, but they are not so difficult to
understand. After you locate the details to certain sentences in the passage, try to identify the
reasoning, and then turn to the answer choices. Fortunately, once you identify the reasoning, you
will quickly get the right answer since there are great differences among these five choices.

B. Assumption/Weaken/Strengthen

Weaken, Support, and Assumption are the other three types of question you are expected to
solve in critical reasoning question. As we said above, you need to evaluate the argument and
identify the assumptions. Typical questions would be:

z Which of the following, if true, would most weaken the theory proposed by Snyder et al?

z Which of the following, if true, would most strongly support Keyssar's findings as they are
described by the author?

Sample Question

The cutting-edge science is ringing alarm bells. Avian flu virus picked up by pigs can swap
genetic materials with another flu virus already in the pig and become a new, hitherto
unknown flu virus for which no person, no animal has preexisting immunity. The kind of virus
causes a pandemic because it spreads from human to human.

If you took a peek into history, it turns out that previous influenza pandemics have similar
scenarios. The greatest influenza pandemic in 1918 caused more than 20 million deaths of
soldiers stationed in France. The last influenza pandemic was in 1968, known as the Hong
Kong flu (H3N2). Thousands of deaths and millions were infected worldwide.

39
The other examples are the Nipah virus and Japanese Encephalitis virus, which find pigs to
be good hosts. With JE, the virus circulates in the blood of infected pigs. When infected pigs
are bitten by Culex mosquitoes, the virus replicates in the mosquito's gut. The next time the
mosquito bites a human, the virus is passed on. The pig doesn't get sick as such. The Nipah
virus causes pneumonia symptoms in pigs. In humans, it causes encephalitis, and humans
catch it only with direct contact with infected pigs. Symptoms range from mild headache to
permanent brain damage, and can be fatal.

It's merely a phenomenon of nature that the pig is the "mixing vessel" for the new germ. But
make no mistake, the pig is not the villain, neither is the chicken. It's actually us, and our
horrible farm practices, outdated agricultural policy and, most of all, reckless disregard of our
ecology and environment. "Hygiene and management can control what eventually happens,"
says Lam. "Good farming practice will prevent serious outbreaks and infection to humans."
Despite knowing that, animal diseases and the possibility of transmission to humans are
becoming quite alarming. Of the 35 new emerging diseases in the last 20 years, more than
70 per cent involved animals.

In fact, what we may have done is unwittingly create the perfect launch pad for an influenza
pandemic that will likely kill large numbers of people across the globe. Although scientists
say it's impossible to predict the odds that the virus will alter its genetic form radically enough
to start leaping from human to human, the longer H5N1 is out there killing chickens, the
higher the chances are.

All of the following situations are similar to the spread of avian flu virus described in the first
paragraph EXCEPT:

(A) The BT2 spread from a pig to another pig, and thus causes significant disease in pig.

(B) The AIDS viruses transferred from monkeys to man and spread across the world.

(C) The SARS virus originates from some wildlife and is picked up by civet cats from which
humans got it.

(D) Nipah virus circulates in the blood of infected pig, which is bitten by Culex mosquitoes, the
virus replicates in the mosquito's gut. The next time the mosquito bites a human, the virus
is passed on.

(E) H5N1 starts in chickens and leaps from human to human.

The question requires you to recognize a situation that is not similar to the spear of avian flu.
Before considering following answer choices, we fist define its rationale. It is something like this:
Avian flu virus picked up by pigs and is transferred to human. All of the situations described in the
answer choices are similar to it ex that in choice A (from animal to animal). Therefore, A is the
best answer.

40
5. Difficult-to-locate Question

Some question does not ask for the central idea of a passage. Rather, it requires you to draw a
conclusion based on the passage:

z According to the passage, which of the following is the author most likely to agree with?

z The passage supplies information that would answer which of the following questions?

Unlike Recall Question or Inference Question, Difficult-to-locate Question does not contain key
words that you can use to locate the details tested. In order to solve this type of question, you
have to skim through the passage again and again until you get the right answer. Eliminating
wrong choices often take considerable time since the answer choices are often too long and
complicated to understand. That is why most test takers regard difficult-to-locate question as the
most difficult one in reading comprehension. The good news is that if you encounter several
questions like these, then you probably get a high score since questions are presented based on
your performance on the previous questions.

Sample Question

Indian firms have achieved the highest levels of efficiency in the world software outsourcing
industry. Some researchers have assumed that Indian firms use the same programming
languages and techniques as Chinese firms but have benefited from their familiarity with
English, the language used to write software code. However, if this were true, then one
would expect software vendors in Hong Kong, where most people speak English, to perform
not worse than do Indian vendors. However, this is obviously not the case.

Other researchers link high Indian productivity to higher levels of human resource investment
per engineer. But a historical perspective leads to a different conclusion. When the two top
Indian vendors matched and then doubled Chinese productivity levels in the mid-eighties,
human resource investment per employee was comparable to that of Chinese vendors.
Furthermore, by the late eighties, the amount of fixed assets required to develop one
software package was roughly equivalent in India and in the China. Since human resource
investment was not higher in India, it had to be other factors that led to higher productivity.

A more fruitful explanation may lie with Indian strategic approach in outsourcing. Indian
software vendors did not simply seek outsourced contract more effectively: they made
aggressive strategic in outsourcing. For instance, most software firms of India were initially
set up to outsource the contract in western countries, such as United States. By contrary,
most Chinese firms seem to position their business in China, a promising yet
under-developed market. However, rampant piracy in China took almost 90 percents of
potential market, making it impossible for most Chinese firms to obtain sufficient

41
compensation for the investment on development and research, let alone thrive in
competitive environment.

According to the passage, which of the following statements is true of Indian software developers?

(A) Their productivity levels did not equal those of Chinese software engineers until the late
eighties.

(B) Their high efficiency levels are a direct result of English language familiarity.

(C) They develop component-specific software.

(D) They are built to outsource the western orders.

(E) They develop more packages of software than do those in Chinese developers.

In the middle of the last paragraph, the author states that For instance, most software firms of
India were initially set up to outsource the contract in western countries, such as United
States. Thus, the best answer is D.

42
Section 6: Six test points

While four-step procedure helps you to understand a passage and the five types of question
guide you how ETS test the understanding of the passage, the six test points will in advance
introduce what would be tested even before you read the questions. As you are reading the
passage, keep alarm to certain words or phrases since they would later act as clues for
answering the following questions. We call these signal words or phrases as test points.

In the following passage, we will introduce you the six most common test points in reading
comprehension. Once you become familiar with these test points, you will get advantage in speed
to come up with the right answer choice.

1. Comparison

Words or phrases: like, unlike, in contrast to, similarly

Question Type: recall question, inference question

Here is an example:

The fact that reducing price can generate a competitive advantage for a company does not
mean that every reduction in price will create such an advantage. Price reduction, like
improvement in service, must be balanced against other types of efforts on the basis of
direct, tangible benefits such as increased revenues. If a company is already effectively on a
par with its competitors because it provides product at an acceptable price and keeps
customers from leaving at an unacceptable rate, then reduction in price may not be effective,
since price is not necessarily the deciding factor for any customer in any situation.

This truth was not apparent to managers of one operating system software vendor, which
failed to improve its competitive position despite its attempt to reduce price. The software
managers did not recognize the level of customer inertia that arises from the inconvenience
of switching operating system. Nor did they analyze their reduction in price to determine
whether it would attract new customers by producing a new standard of price that would
excite customers or by proving difficult for competitors to copy.

Sample question

According to the passage, reduction in price are comparable to improvement in service in


terms of the

(A) tangibility of the benefits that they tend to confer

(B) increased revenues that they ultimately produce

43
(C) basis on which they need to be weighed

(D) insufficient analysis that managers devote to them

(E) degree of competitive advantage that they are likely to provide

To answer this question, first locate the question to the second sentence of the passage. "Price
reduction, like improvement in service, must be balanced against other types of efforts on the
basis of direct, tangible benefits such as increased revenues." In other words, they are
comparable based on which they need to be weighed. Therefore, C is the correct answer.

44
2. Example & Listing

Words or phrase: such as, as well as, for example, for instance

Question type: Listing, Exampling

Let's look at a sample question for the same passage.

The fact that reducing price can generate a competitive advantage for a company does not
mean that every reduction in price will create such an advantage. Price reduction, like
improvement in service, must be balanced against other types of efforts on the basis of direct,
tangible benefits such as increased revenues. If a company is already effectively on a par
with its competitors because it provides product at an acceptable price and keeps customers
from leaving at an unacceptable rate, then reduction in price may not be effective, since price
is not necessarily the deciding factor for any customer in any situation.

This truth was not apparent to managers of one operating system software vendor,
which failed to improve its competitive position despite its attempt to reduce price. The
software managers did not recognize the level of customer inertia that arises from the
inconvenience of switching operating system. Nor did they analyze their reduction in price to
determine whether it would attract new customers by producing a new standard of price that
would excite customers or by proving difficult for competitors to copy.

The discussion of the operating system software vendor last paragraph serves which of the
following functions within the passage as a whole?

(A) It describes an exceptional case in which reduction in price actually failed to produce a
competitive advantage.

(B) It illustrates the pitfalls of choosing to reduce price at a time when business strategy is
needed more urgently in another area.

(C) It demonstrates the kind of analysis that managers apply when they choose one kind of
business strategy over another

(D) It supports the argument that strategies in certain aspects are more advantageous than
strategies in other aspects.

(E) It provides an example of the point about reduction in price made in the first paragraph.

Clearly, the author intends to prove his position that reduction in price does not necessarily
generate competitive advantage. E is the correct.

45
3. People, Date & Place

Phrase: in the nineteenth-century, prior to mid-1970's, Snyder proposed that.

Question: inference question, main idea question

Indian firms have achieved the highest levels of efficiency in the world software outsourcing
industry. Some researchers have assumed that Indian firms use the same programming
languages and techniques as Chinese firms but have benefited from their familiarity with
English, the language used to write software code. However, if this were true, then one
would expect software vendors in Hong Kong, where most people speak English, to perform
not worse than do Indian vendors. However, this is obviously not the case.

Other researchers link high Indian productivity to higher levels of human resource investment
per engineer. But a historical perspective leads to a different conclusion. When the two top
Indian vendors matched and then doubled Chinese productivity levels in the mid-eighties,
human resource investment per employee was comparable to that of Chinese vendors.
Furthermore, by the late eighties, the amount of fixed assets required to develop one
software package was roughly equivalent in India and in the China. Since human resource
investment was not higher in India, it had to be other factors that led to higher productivity.

A more fruitful explanation may lie with Indian strategic approach in outsourcing. Indian
software vendors did not simply seek outsourced contract more effectively: they made
aggressive strategic in outsourcing. For instance, most software firms of India were initially
set up to outsource the contract in western countries, such as United States. By contrary,
most Chinese firms seem to position their business in China, a promising yet
under-developed market. However, rampant piracy in China took almost 90 percents of
potential market, making it impossible for most Chinese firms to obtain sufficient
compensation for the investment on development and research, let alone thrive in
competitive environment.

The author suggests that if the researchers of India mentioned in paragraph 1 were correct, which
of the following would be the case?

(A) The computer used in India software firms would be different from the computer used in
China firms.

(B) Indian engineers would be trained to do several different programming jobs.

(C) Familiarity with English language would not have an influence on the productivity levels of
engineers.

(D) The engineers in India-run firms would have lower productivity levels if they have a poor
command of English.

46
(E) The production levels of India-run firms located in the China would be equal to those of
firms run by China firms.

If the researchers are correct, then the familiarity with English determines the productivity of
engineers. That is, if the engineers in India-run firms have a poor command of English they would
have lower productivity levels, as stated in choice D.

47
4. Words of Attitude and Transition

Word: Correctly, qualified, do (does, did), may (might), correctly, first (second, third)

Question Type: Tone Question, Main Topic Question, Structure Question

The fact that reducing price can generate a competitive advantage for a company does not
mean that every reduction in price will create such an advantage. Price reduction, like
improvement in service, must be balanced against other types of efforts on the basis of direct,
tangible benefits such as increased revenues. If a company is already effectively on a par
with its competitors because it provides product at an acceptable price and keeps customers
from leaving at an unacceptable rate, then reduction in price may not be effective, since price
is not necessarily the deciding factor for any customer in any situation.

This truth was not apparent to managers of one operating system software vendor, which
failed to improve its competitive position despite its attempt to reduce price. The software
managers did not recognize the level of customer inertia that arises from the inconvenience
of switching operating system. Nor did they analyze their reduction in price to determine
whether it would attract new customers by producing a new standard of price that would
excite customers or by proving difficult for competitors to copy.

The passage suggests that operating system software managers failed to consider whether or not
the price reduction mentioned last sentence

(A) was too complicated to be easily described to prospective customers

(B) made a measurable change in the experiences of customers purchasing

(C) could be sustained if the number of customers increased significantly

(D) was an innovation that competing vendors could have imitated

(E) was adequate to bring the vendors general level of price to a level that was comparable
with that of its competitors

The passage following failed to describes the failure. The best choice is D, which is stated in the
last sentence.

48
5. Counter-evidence Indicators

Counter-evidence words warn that the author is about to either make a U-turn or introduce a
counter-premise (concession to a minor point that weakens the argument).

But Although However Yet

Despite Nevertheless Nonetheless Except

In contrast Even though

Counter-evidence words mark natural places for questions to be drawn. At a pivotal word, the
author changes direction. The GMAT writers form questions at these junctures to test whether
you turned with the author or you continued to go straight. Rarely do the GMAT writers let a
pivotal word pass without drawing a question from its sentence. As you read a passage, note
the pivotal words and refer to them when answering the questions.

Example

China as a nation faces two major financial problems. First, eighty-four percent of
state-owned enterprises do not generate profit. Government failed to make money from such
business. Rather, it has to appropriate substantial funds to these enterprises in order to
prevent them from going bankrupt and thus resulting in high unemployment rate. Second,
203 million of civilians in countryside will not be able to gain pension after they retire due to
the limited budget of government.

I would like to make an outrageous suggestion that would at one stroke generate finance
earnings and provide funds for civilians retirement. I would propose that government sells its
holdings in state-owned enterprises on the open market. Such sales would provide
substantial funds for village civilians pension. At the same time, they could cut down
financial burden on these state-owned enterprises.

You might object that government would be deprived of the opportunity to share its
enterprises profit if someday they make money. I agree. Sell holdings of enterprises that
would never generate profit. But, you might reply, every enterprise that competes on the
market has potential. Here we part company. Theoretically, you may be correct in claiming
that every enterprise has the potential to make money. Practically, you are wrong.

I refer to the thousands of state-owned enterprises that are not likely to make money. These
companies are 100 percent held by the nation as a whole. Government officials are
appointed as the chairman, CEO and president. The management was not responsible for
the public interest, but for the nation as a whole. If there is no significant loss in business,

49
they will soon be promoted back to the higher level position in government. If their
companies perform great, these executives receive direct money compensation. However,
their salary, when combined with such compensation, will be much less than the amount
they would earn if were in private company.

It would be unrealistic to suggest that village civilians would have sufficient funds if
governments shares were sold on the open market. But the demand for compensating the
state-own enterprises would be substantially reduced.

The author anticipates which of the following initial objections to the adoption of his proposal?

(A) Government will not be able to sell its holdings with state-owned enterprise.

(B) The ability of governments to control the national economy will be weakened if
state-owned enterprises are sold to private owners.

(C) It is impossible to find enterprises that will never generate profit.

(D) The poor performance of state-owned enterprises will continue.

(E) The countryside civilians are sill unable to seek financial support from government.

In the third paragraph, the author illustrates a possible objection to his proposal. The opposite
views are government would be deprived of the opportunity to share its enterprises profit
if someday they make money and every enterprise that competes on the market has
potential. Choice C presents one of them, and is the correct answer.

50
6. Special Punctuation

Punctuation: Quotation, Parentheses, Dash

Question type: Recall question, Inference Question

The cutting-edge science is ringing alarm bells. Avian flu virus picked up by pigs can swap
genetic materials with another flu virus already in the pig and become a new, hitherto
unknown flu virus for which no person, no animal has preexisting immunity. The kind of virus
causes a pandemic because it spreads from human to human.

If you took a peek into history, it turns out that previous influenza pandemics have similar
scenarios. The greatest influenza pandemic in 1918 caused more than 20 million deaths of
soldiers stationed in France. The last influenza pandemic was in 1968, known as the Hong
Kong flu (H3N2). Thousands of deaths and millions were infected worldwide.

The other examples are the Nipah virus and Japanese Encephalitis virus, which find pigs to
be good hosts. With JE, the virus circulates in the blood of infected pigs. When infected pigs
are bitten by Culex mosquitoes, the virus replicates in the mosquito's gut. The next time the
mosquito bites a human, the virus is passed on. The pig doesn't get sick as such. The Nipah
virus causes pneumonia symptoms in pigs. In humans, it causes encephalitis, and humans
catch it only with direct contact with infected pigs. Symptoms range from mild headache to
permanent brain damage, and can be fatal.

It's merely a phenomenon of nature that the pig is the "mixing vessel" for the new germ. But
make no mistake, the pig is not the villain, neither is the chicken. It's actually us, and our
horrible farm practices, outdated agricultural policy and, most of all, reckless disregard of our
ecology and environment. "Hygiene and management can control what eventually happens,"
says Lam. "Good farming practice will prevent serious outbreaks and infection to humans."
Despite knowing that, animal diseases and the possibility of transmission to humans are
becoming quite alarming. Of the 35 new emerging diseases in the last 20 years, more than
70 per cent involved animals.

In fact, what we may have done is unwittingly create the perfect launch pad for an influenza
pandemic that will likely kill large numbers of people across the globe. Although scientists
say it's impossible to predict the odds that the virus will alter its genetic form radically enough
to start leaping from human to human, the longer H5N1 is out there killing chickens, the
higher the chances are.

What does the author mean by describing the pig as mixing vessel?

(A) Pig is the place where various viruses reside.

51
(B) Pig is the pot in which viruses swap genes and become new, deadly germs.

(C) Viruses are mixed inside the body of pig.

(D) New germs come to the body of pig and reside there.

(E) Pig attracts viruses.

The question requires you to determine the meanings of mixing vessel. At the beginning of the
passage, the author states that Avian flu virus picked up by pigs can swap genetic materials with
another flu virus already in the pig and become a new, hitherto unknown flu virus for which
no person, no animal has preexisting immunity. The kind of virus causes a pandemic because it
spreads from human to human. In other words, pig is the pot in which viruses swap genes and
become new, deadly germs. Therefore, the correct answer is B.

Review

One principal

Don't rely on memory or on daily life experiences. Answer all the questions on the basis of what is
stated or implied in the passage.

Two Styles

Presentation is to deliver an idea that the author will agree or partially agree. Argumentation is to
develop two ideas or systems and then point out why one is better than the other (contrasting) or
just simply refute both of them and developed the author's own idea.

Three Subjects

There are three major subjects that a typical GMAT reading passage may discuss about: natural
science, social science and business subject.

Four-step Procedure

Use the four-step process to read a passage. First dissect the first paragraph, then skim the
passage, get the main idea and diagram the organization of the passage.

Five Types of Question

There are five types of question you may encounter in real GMAT test: main idea question, recall
question, inference question, critical reasoning question and the difficult-to-locate question.

Six Test Points

The most common six test points are comparison, exemplifying, people, special punctuation,
counter-evidence and mood words.

52
Chapter 2 Sentence Correction

Introduction

Sentence Correction on the test day

About 17 of the 41 Verbal section questions are Sentence Correction. And in the first 10 question,
there are about 5 sentence correction question. This is why Sentence Correction is so important
to your GMAT performance. The directions on the test day for this part look like this:

Directions: The following questions consist of sentences that are either partly or entirely
underlined. Below each sentence are five versions of the underlined portion of the sentence.
Choice (A) is a copy of the original version. The four other answer choices change the underlined
portion of the sentence. Read the sentence and the five choices carefully and select the best
version.

You are expected to pick up one choice that will make the whole sentence clear, concise and free
with grammar errors. Choice A is always the same as the original underlined, so you only have to
read the other four choices.

What does the Sentence Correction test?

The Sentence Correction section tests the correctness of English grammar, including
subject-verb agreement, modification, pronouns, idioms, and etc. The GMAT also tests the
effectiveness of expression, such as parallelism, and logicality. Therefore, the best answer should
be both exact and clear, without ambiguity, redundancy, or awkwardness.

The GMAT does not test mechanics concerns, such as punctuation, capitalization, misspelling,
etc. You should never spend time on figuring out whether a word is misspelled, or whether a
comma is the right punctuation here. Rather, you should focus on the grammar usage and
standard written English.

Now, let's look at an example to see what a sentence correction question could be.

Satisfied by the strong performance on GMAT test, it was decided by Peter to give himself a
two-week rest.

A. it was decided by Peter to give himself a two-week rest


B. Peter decided to give himself a two-week rest
C. a two-week rest was given by Peter to himself

53
D. Peters decision was to give himself a two-week rest
E. it was decided that Peter give himself a two-week rest

Three-step method

A. Read the complete sentence

Read the whole sentence to get an idea about the basic meanings. Dont read only the underlined
part of the sentence. If you simply read the underlined, you are probably following the trap set by
the test maker.

Example

Satisfied by the strong performance on GMAT test, it was decided by Peter to give himself a
two-week rest.

B. Figure out what the question is testing

Before you read the answer choices, spend several seconds on figuring out what the question is
testing. If you have no idea, you may look at the different answer choices and see what the
changes have been made, so as to figure out the possible errors in the sentence.

Satisfied by the strong performance on GMAT test, it was decided by Peter to give himself a
two-week rest.

A. it was decided by Peter to give himself a two-week rest


B. Peter decided to give himself a two-week rest
C. a two-week rest was given by Peter to himself
D. Peters decision was to give himself a two-week rest
E. it was decided that Peter give himself a two-week rest

C. Eliminate wrong choices

The process of elimination is extremely useful in Sentence Correction section. Eliminate it once
you find one error, until you are left with only one or two choices. If you are unable to eliminate
four choices, take a second look at the original sentence. Some questions test more than one
grammar knowledge. If you still fail, you may just make a guess.

Satisfied by the strong performance on GMAT test, it was decided by Peter to give himself a
two-week rest.

A. it was decided by Peter to give himself a two-week rest

54
B. Peter decided to give himself a two-week rest
C. a two-week rest was given by Peter to himself
D. Peters decision was to give himself a two-week rest
E. it was decided that Peter give himself a two-week rest

The correct answer is B because it is Peter who was satisfied. In choice B, Peter appears as the
subject. Choices A, C, D and E are incorrect because they used it, a two-week rest, Peters
decision, and it as the sentence subject. In fact, the participial phrase beginning with satisfied
should modify the subject of the main clause in order to follow the grammar rule.

Fourteen types of errors

Based on our research, the following 14 types of errors represent 96% of Sentence Correction
questions. You should become sensitive to these errors.

55
Section 1: Subject-Verb Agreement

The subject and verb must agree both in person and number. If the subject is plural, then the verb
must be plural too. If the subject is singular, then the verb must also be singular.

Intervening modifier has no effect on subject-verb agreement.

If a singular subject is accompanied by phrases or clauses, it remains singular. Subject that


contains and is plural. The subject that contains or can be either plural or singular depending
on the number of the last item.

When the subject and verb are reversed, they still must agree in both number and person.

Note:

1) The following nouns are singular: work, happiness, poverty, honesty, faith, time, visibilitymilk,
tea, cotton, petrol
2) Singular and plural nouns with same form: bison, cattle, sheep, fish, aircraft, means, series,
Chinese, Japanese
3) Subject nouns ending with ics are singular: economics, statistics,
4) Verb phrases as a subject is singular. Preparing for the TOEFL is not an easy task.

Example

Since 1999, the number of internet websites with the domain name ending with .com have grown
from 62 million to nearly 78 million.

A. have grown from 62 million to nearly 78 million

B. are growing from 62 million to nearly 78 million

C. grew from 62 million to nearly 78 million

D. grow from 62 million to nearly 78 million

E. has grown from 62 million to nearly 78 million

E is the correct answer. In choice A, the plural verb have does not agree with the singular subject
number. Choices B and D commits the same fallacy by using plural verbal phrase are growing
and grow respectively. B, C, and D also misused the verbal tense which should be present
perfect.

56
Section 2: Verb Time Sequences

Faulty verb tense is common on GMAT test. But it is easy to solve if you become familiar with
verb tense rules. A verb has four principal parts: present tense, past tense, past participle and
present participle. The present tense is used to express present tense or general truths. The past
tense is used to express past tense.

The past participle is used to form the present perfect tense, past perfect tense, or future perfect
tense. The present participle is used to form the present progressive tense, the past progressive
tense, or the future progressive tense.

Keep alert when several verbs appear. If several events happened at different times, choose one
as the basic in time sequence.

Eighty percent of notebook computers that were sold in United States last year were
manufactured in China, a country that has the largest population in the world.

A. Eighty percent of notebook computers that were sold in United States last year were

B. Eighty percent of notebook computers that were sold in United States last year had been

C. Eighty percent of notebook computers that were sold in United States last year have been

D. Last year eighty percent of notebook computers were sold in United States that have been

E. Last year eighty percent of notebook computers that were sold in United States had been

In using only one verb tense, were, choice A fails to indicate that the computers were
manufactured before sold. Choices C and D use the present perfect tense incorrectly, saying in
effect that the computers have been manufactured after they were sold last year. Choice E
suggests that the manufacturing of the notebook computers, rather than selling, occurred last
year, thus making the sequence of events unclear. Only B uses verb tenses correctly to indicate
that manufacturing of the computers was completed prior to the selling.

57
Section 3: Modification

In GMAT grammar, a modifier can be an adjective, adverb, phrase, an appositive or even a


clause. In the following passage, we will introduce each of them. As a general rule, a modifier
should be placed as close as possible to what it modifies.

A. Adjective or adverb as a modifier

An adjective can modify a noun, but an adverb can not. An adverb is often used to describe the
extent or degree of an adjective or a verb.

Example

According to a report from Anderson Accounting, the gross sales of General Movies in 2002 were
$86 millions as many as their expected revenues.
A. as many as their expected
B. more than their expected
C. as many as their excepted
D. more than their expectedly
E. as many as their expectedly

Choices A, C, and E do not state the comparison logically. The expression as many as indicates
equality of quantity, but the sentence indicates that the gross sales exceed the expected
revenues by $86 millions. In B, the best choice, more than makes this point of comparison clear.
B also correctly uses the adjective expected, rather than the adverb expectedly used in D and E,
to modify the noun phrase revenues.

B. Clause as a modifier

A clause beginning with that, which or where modifies the words or phrases nearest to it. Here,
"which" initiates a clause modifier and can not be used to refer to a sentence.

Example

The current downturn in the U.S. economy is encouraging many young professionals to return to
school, which doubles to twice the number of applicants five years ago.

A. which doubles to twice the number of applicants five years ago

B. doubling to twice the number of applicants five years ago

C. which doubles to twice the number of applicants that were five years ago

58
D. doubling to twice the number of applicants five years before

E. which doubles to twice the number of applicants five years before

The pronoun which should be used to refer to a previously mentioned noun, not to the idea
expressed in an entire clause. In A, C, and E, which seems to refer to a vague concept involving
young professionals returning to school, but there is no specific noun, such as return, to which it
can refer. B and D use the correct participial form, doubling, to modify the preceding clause, but D,
like A, uses five years before rather than five years ago, a phrase that is more idiomatic in context.
B, therefore, is the best answer.

C. A long phrase as a modifier

When a long phrase initiates a sentence, make sure that it modifies the subject of the sentence.

Example

Using the KB833330, a new virus known as Bagle can be blocked outside the Local Area
Network.

A. Using the KB833330, a new virus known as Bagle can be blocked outside the Local Area
Network.

B. A new virus known as Bagle can be blocked outside the Local Area Network, using the
KB833330.

C. Blocking a new virus known as Bagle outside the Local Area Network, an engineer can use
the KB833330 by an engineer

D. Outside the Local Area Network, a new virus known as Bagle can be blocked using the
KB833330 by an engineer

E. Using the KB833330, an engineer can block a new virus known as Bagle outside the Local
Area Network.

Choice A presents a dangling modifier. The phrase beginning the sentence has no noun that it
can logically modify and hence cannot fit anywhere in the sentence and make sense. Coming first,
it modifies a new virus, the nearest free noun in the main clause; that is, choice A says that a
new virus are using the KB83330. Choice B contains the same main clause and dangling modifier,
now at the end. Contrary to intent, the wording in choice C suggests that engineers can use the
KB833330 after they block a new virus. In choice D the phrase using ... the KB833330 should
follow engineer, the noun it modifies. Choice E is best.

59
D. Appositive as a modifier

Like a clause, an appositive can also serve as a modifier. In most cases, appositive is separated
by a comma. The appositive should be in the same number as modifiee.

Example

The GMAT math section consists of 37 questions, each question a test of a certain math concept.
A. each question a test on a certain math concept
B. all the questions a test on a certain math concept
C. all the questions are tested on a certain math concept
D. every question is tested on a certain math concept
E. each question is tested on a certain math concept

Choice A is best: the appositive terms question and test, both singular, agree in number; both
also agree with rule on sentence structure. In C, D, and E choices, Runs-on sentence is
committed.

60
Section 4: Parallelism

Similar elements must be expressed in similar form: all nouns, all infinitives, all gerunds, all
prepositional phrases, or all clauses must agree. Test writers often use a parallel structure for
dissimilar elements. In the case you are not sure which form should be used, the form of the
second element of the series determines the form of all subsequent elements.

Example

According to a survey, a company president typically spends 60 percent of his or her time on
communicational activities, such as answering the calls, communicating with clients and to speak
on the meetings.

A. communicating with clients and to speak

B. communicating with clients and speaking

C. to communicate with clients and speak

D. to communicate with clients and to speak

E. to communicate with clients and speaking

Because the verb phrases used to describe the communicational duties are governed by the
phrase communicational duties such as, they should each be expressed in the present
participial (or "-ing") form to parallel answering. Choices A, C, D, and E all violate parallelism by
employing infinitives (to...) in place of participial phrases. Only B, the best answer, preserves the
sense of the original, uses the correct idiom, and observes the parallelism required among and
within the three main verb phrases.

61
Section 5: Pronoun

Pronouns must agree with their antecedents in both number and person.
Subject pronoun must be used as a subject. Object pronoun must be used as an object.
A pronoun must clearly stand for a noun. If a pronoun follows two nouns, it is often unclear
which of the nouns the pronoun refers to. This is the most common error on the GMAT.
The word "which" introduces non-essential clauses and "that" introduces essential clauses.
"Who" refers to individuals; "that" refers to a group of persons, class, type, or species.

Subject Object Possessive Possessive Self Singular


(adjective) (nouns) or Plural
I Me my mine myself Singular
We Us our ours ourselves Plural
You You your yours yourself Singular
You You your yours yourself Plural
He Him his his himself Singular
She Her her hers herself Singular
It It its its itself Singular
they Them their theirs themselves plural

Example

The best way for an IT professional to protect data is to periodically back it in a pre-formatted disc.

A. to periodically back it in a pre-formatted disc

B. if it is quickly backed in a pre-formatted disc

C. for it to be backed periodically in a pre-formatted disc

D. if the data is periodically backed in a pre-formatted disc

E. to have them periodically backed in a pre-formatted disc

For parallelism, the linking verb is should link two infinitives: The only way to salvage ... is to
back. Choice A begins with an infinitive, but the plural pronouns I do not agree with the plural
noun data. Choices B, C, and D do not begin with an infinitive, and all present pronoun errors: the
singular pronouns cannot grammatically refer to data. The best choice, E, has parallel infinitives.

62
Section 6: Comparisons

The comparison should be both logically similar and grammatically parallel. There are typically
three types of comparisons: quality comparison, quantity comparison and analogy.

Types of Words of Comparison Indicators of


Comparison Comparison

Quality like/unlike in contrast to rather/other that/those of,


than was/is/were/are,

did/do/does
Quantity less/more than as much as the same as

Analogy just as so too so as

1. Quality Comparison

If the comparison begins with like, unlike, rather than, in contrast to, not but, it is a quality
comparison. This comparison is common, but easy in sentence correction. All you have to do is to
check if the item following the word of comparison is comparable to the subject of main sentence.

Example

Unlike a corporation, which pays tax based on its related revenues, a fixed amount of tax is paid
by a sole ownership business.

A. a fixed amount of tax is paid by a sole ownership business

B. with a sole ownership business a fixed amount of tax is paid

C. a sole ownership is paid a fixed amount of tax

D. for a sole ownership business a fixed amount of tax is paid

E. a sole ownership pays a fixed amount of tax

Choice E, the best answer, correctly uses a parallel construction to draw a logical comparison:
Unlike a corporation,..., a sole ownership business.... Choice A illogically compares a
corporation, an entity, with a fixed amount of tax, money. In choice C, a sole ownership
business can not be paid for tax. Choices B and D are syntactically and logically flawed because
each attempts to compare the noun corporation and a prepositional phrase: with a fixed
amount of tax. Choices B and D are also imprecise and awkward. Finally, choice E is the only
option that supplies an active verb form, pays to parallel pays.

63
2. Quantity Comparison

Typical comparisons in this kind are introduced by such idioms as more/less than, the same as,
as many as. The errors in quantity comparison are more difficult to identify since the sentence
that contains quantity comparison is generally long. The following two procedures can be taken to
tackle quantity comparison:

Check if the idiom or phrase is correct and complete

Check if the introductory word has been presented to initiate the comparison

Example

In addition to having more employees than UT StartCom, the employees in GenericSart are
higher educated than those in UT StartCom, with more graduate students.

A. the employees in GenericSart are higher educated than those in

B. GenericStart has higher educated employees than those do

C. the employees in GenericStart are higher educated than those are in

D. GenericStart employees are higher educated that those are in

E. GenericStart has higher educated employees than

In this sentence, the initial clause modifies the nearest noun, identifying it as the thing being
compared with UT StartCom. By making employees the noun modified, choices A, C, and D
illogically compare UT StartCom with employees and claim that the employees in GenericStart
has higher educated employees than UT StartCom does. B, the best choice, logically compares
UT StartCom to GenericStart by placing the noun GenericStart immediately after the initial
clause. B also uses those to refer to employees in making the comparison between the
employees of UT StartCom and GenericStart. Choice E needs either those in or do after UT
StartCom to make a complete and logical comparison.

3. Analogy

While analogy is a logical concept, it is also used in Sentence Correction to draw a comparison
between two similar things. Like comparison, analogy should also be logical and grammatical.

Example

The gravity will apply the same to an airplane flying in the air as a ship floating on the water.

A. air as a

B. air as to a

64
C. air; just as it would to a

D. air, as it would to the

E. air; just as to the

B, the best choice, uses the idiomatic and grammatically parallel form the same to X as to Y.

65
Section 7: Choice of Word

Sometime, the test-takers try to puzzle you by using some similar words with different meanings.
In choosing a choice, make sure it doesn't contain the wrong word.

Affect/Effect:

Effect is a noun meaning "a result."

Example: Increased fighting will be the effect of the failed peace conference.

Affect is a verb meaning "to influence."

Example: The rain affected their plans for a picnic.

Fewer/Less: (number/amount)

Use fewer when referring to a number of items. Use less when referring to a continuous quantity.

Example: In the past, we had fewer options.

Example: The impact was less than what was expected.

Example

A report by Business Weekly indicated that the number of money invested by companies in
Business and Research in 2003 was twice that in 2002.

A. the number of money invested by companies in Business and Research in 2003 was

B. the number of money invested by companies in Business and Research in 2003 were

C. the number of money invested by companies in Business and Research in 2003 are

D. the amount of money invested by companies in Business and Research in 2003 were

E. the amount of money invested by companies in Business and Research in 2003 was

Choices A, B, and C are flawed because the uncountable noun money should be modified by
amount rather than number. In addition, B, C, and D incorrectly use the plural verb were or are
with the singular noun money. Choice E, the best answer, is both grammatically correct and
concise.

66
Section 8: Idioms

We have summarized most of the idioms tested on GMAT sentence corrections. By memorizing
the following lists, you should be able to solve most the idioms questions on the test day.

act on ability to do according to


account for ask for as far as is known
at least at one time a cluster of
a great deal a number of a minimum of

bare of belong to begin doing


begin to benefit from be able to
be adopted to be associated with be appreciated for
be based on be beneficial to be capable of
be characterized by be composed of be concerned with
be considered to be be credited with doing be dedicated to doing
be depend on be derived from be destined to
be divided into be engaged in be equal to
be expose to be familiar with be famous for
be filled with be forced to do be found in
be full of be inclined to be involved in
be known as be known for be made from
be made of be noted for be obtained from
be originated from be rich in be related to
be resistant to be subjected to be supposed to
be similar to be suited for be typical of
be valuabel for be viewed as be woven from

67
break away from bring about bring to light
by means of

close to change into coincide with


consist of contrary to contribute to
concentrate on convert t into combine with
comment on come into contact with
come from come into contract with
carry out

date back to deal with dedicate to doing


demand for depend on deter sb. from doing
devote to differ from draw from
do no harm to do/deal with due to + n.

excel in/at

feed on focus attention on

68
G

give off give way to give up doing


grant sb. sth.

interest in interfere wit in addition to


in connection with in danger of in history
in nature in honor of in comparison with
in relation to in response to in spite of
in the future

join with

lead to live in look for

make up meet one's goal mingle with


more than

69
N

native to no more than not more than

on account of on behalf of

participate in pay for permit sb. to do


play a key role protect from prohibit..from

range from to rank first among refer to


rely on regardless of rest on
result from run for

settle down sever as share with


shield from so as to specialize in doing
spend doing spread to start doing
start to strive to substitute for
succeed in doing

70
T

take place take charge of tend to


the ratio toof to the use of think of
togeter with to a great extent transform into

use up make use of

vary in

W
warm sb. of sth.

71
Section 9: Sentence Structure

There are limited types of sentence used in English language. The five of them are

z Declarative (simple statement): (Subject first) Mary (then verb) loves (then object) turnips.

z Interrogative (a question): Do you need help identifying a question?


z Imperative (command): Do as I say.
z Exclamatory (exclamation): What exciting it is!

z Conditional (condition expressed): If I pass the exam, I intend to move to Arizona.

Sentences are also classified as simple and complex. Complex sentences contain phrases and
clauses that transform the "I like candy" sentences of your childhood to the more sophisticated
constructions of adult speech. The most common errors for sentence structure is the RUN-ON
sentence, a sentence that lacks of proper conjunctions.

Wrong: There is agreement among United States voters that there is waste in government and
that the government as a whole spends beyond its means, it is difficult to find broad support for a
movement toward a minimal state.

Correct: However much United States voters may agree that there is waste in government and
that the government as a whole spends beyond its means, it is difficult to find broad support for a
movement toward a minimal state.

Example

However much people may agree that there is substantial corruption in Chinese government and
that government does its endeavor, it is difficult to keep the problem under control.

A. However much United States voters may agree that

B. Despite the agreement among people to the fact

C. Although people agree

D. Even though people may agree

E. There is agreement among people that

A is the best choice. Choices B, C, and D incorrectly omit that after agree; that is needed to
create the parallel construction agree that there is substantial corruption . . . and that the
government... . Choice E, though it retains that, is grammatically incorrect: because E starts with
an independent rather than a subordinate clause and separates its two independent clauses with

72
a comma, it creates a run-on sentence with no logical connection established between the halves.
In B, the agreement ... to the fact is unidiomatic, and B, C, and E alter the sense of the original
sentence by saying that voters agree rather than that they may agree.

73
Section 10: Subjunctive Mood

When using a subjective verb, the verb following the clause should be in present tone. Also, the
conjunction "that" should not be removed. The most common subjunctive verbs include:

require
demand
request
suggest
rule

Example

Given that most Misubishi cars on the road have some problems with tyre, the Misubishi
Company yielded to hundreds of unsatisfied buyers requesting that it should call back the cars on
the market.

A. requesting that it should

B. requesting it to

C and their request to

D. who requested that it

E. who request it to

Choice D, the best answer, uses the grammatically correct expression requested that it call
back, in which requested that it is followed by the subjunctive verb call. Choice A incorrectly
uses should call rather than call: requesting that already conveys the idea of "should," and at
any rate a modal auxiliary verb, such as should or must, cannot grammatically follow the
expression requested that. Similarly, B and E use the ungrammatical expression
requesting/requested it to. In C, the expression yielded to... buyers and their demand to
call... unnecessarily states that the company yielded to the buyers as well as to their request.
This expression also fails to specify that the company is expected to call back the cars on the
market.

74
Section 11: Ambiguity

Some answer choices may be correct in grammar, but not the right one. If you are unable to
eliminate four choices according to grammar usage, you should look at whether it expresses
clearly without ambiguity. Pay special attention to choices that contain "if" since this conjunction
has two meanings:

1. In the event that: If you get well prepared for GMAT, you will score higher.

2. Whether: I am unsure if you take the GMAT.

So, always avoid the usage of if when it serves as the meaning of whether.

Example

Opposites of privatization in China consider state-owned business to be an integral part of


national treasury and question if privatization could weaken the ability of the country to adjust
national economy.

A. to be an integral part of the criminal justice system and question if

B. as an integral part of the national treasure and they question if

C. as being an integral part of the national treasure and question whether

D. integral part of the national treasure and question whether

E. are an integral part of the national treasure, and they question whether

When consider means "regard as," as it does in this sentence, its object should be followed
immediately by the phrase that identifies or describes that object. Thus, to be in A, as in B, and as
being in C produce unidiomatic constructions in the context of the sentence. Also, although (/and
whether can be used interchangeably after some verbs, question if, which appears in A and B, is
unidiomatic, and they in B is unnecessary. E also contains the unnecessary they, and it uses the
ungrammatical construction consider... facilities are. Grammatically and idiomatically, sound D is
the best choice.

75
Section 12: Redundancy

As I said at the beginning of this Chapter, Sentence Correction not only tests your knowledge of
correct grammatical usage, but also your sense of clear English writing. Your selected answer
should express the intended meaning of the original sentence as precisely as possible, while
avoiding unnecessarily wordy constructions.

The following phrases illustrate typical wordy construction you will encounter in some fluff answer
choices:

Wordy: more than or as much as

Concise: at least as much as

Wordy: a noun that is (are) adj.

Concise: an adj. noun

Example

At least as much as 204 millions or more people in the world earn fewer than $10 per day.
A. At least as much as 204 millions or more people in the world earn fewer than
B. At least as much as 204 millions or more people in the world earn less than
C. More than 204 millions people in the world earn fewer than
D. More than 204 millions people in the world earn less than
E. There are at least 204 millions or more people earn less than

D, the best choice, is idiomatic, clear, and concise. Both A and B incorrectly use much rather than
many to describe the countable noun others. Even if this error were corrected, though, A and B
would still be wrong. Because more than x necessarily includes the sense of at least as many as x.
it is redundant and confusing to use elements of both expressions to refer to the same number of
people. In A and C, fewer is misused.

76
Section 13: Awkward

Sometime, you will find the underlined difficult to understand. You can't tell which part of the
underlined has problem. Instead, you just feel A would not be the right choice. Here, you should
consider whether the expression is awkward. The following expression is awkward, and will never
be the right choice.

A long phrase as a modifier while without a comma as a proper pause.

There be + abstract noun (such as conversion, relation).

Example

Linguists conclude that the more the non-Chinese use Chinese, their intellectual advantage is
greater in skills underlying spatial arranging ability.

A. their intellectual advantage is greater in skills underlying spatial arranging ability.

B. their intellectual advantage is the greater in skills underlaying spatial arranging ability.

C. the greater their intellectual advantage in skills underlying spatial arranging ability.

D. in skills that underlay spatial arranging ability, their intellectual advantage is the greater

E. in skills underlying spatial arranging ability, the greater intellectual advantage is theirs

The best choice is C. The phrase the more the non-Chinese should be completed by a parallel
phrase that begins with a comparative adjective and a noun phrase, as in the greater their...
advantage. Only C correctly completes the structure with a parallel phrase. Choices A. B, D, and
E present structures that are unwieldy and awkward in addition to being nonparallel, and that state
the relationship between language use and skills development less clearly than C does. Also,
underlaying in B and underlay in D are incorrect; the meaning of this sentence requires the
present participle of "underlie," underlying, as a modifier of skills.

77
Section 14: Logicality

Like the rule of comparison, you can not say psychopath is a person who. The proper
expression should be psychopath refers to someone who.

Example
In 2003, the term SARS went deep into everyones mind in South-eastern Asian Area, and in
pathology it is severe acute respiratory syndrome.
A. it is severe acute respiratory syndrome
B. it is someone suffered from severe acute respiratory syndrome
C. they are severe acute respiratory syndrome
D. it refers to severe acute respiratory syndrome
E. it is in reference to severe acute respiratory syndrome

Answer: D

In choices A and B, the pronoun it simultaneously refers forward to severe acute respiratory
syndrome and backward to the term "SARS" As a result, the sentence asserts illogically that the
term is actually a kind of illness rather than a word referring to a kind of illness. Choice C repeats
this fault and adds an error in agreement: they (plural) does not agree in number with the term
(singular). E omits a main verb, such as applied, that, in grammatical context here, is required
after is. In choice D, the best answer, the verb refers is correctly used after it, and the
alignment of pronouns and antecedents is both logical and grammatical.

78
Summary: Finding an error step-by-step in Sentence Correction

With the above fourteen major errors in mind, you are able to eliminate the incorrect choices in a
typical sentence correction question. Below, I will introduce the step-by-step procedure that will
help identify whether the underlined follows the rule of Standard English expression.

Step 1: Is the original sentence correct in the sentence structure? Pay attention to the Runs-on
Sentence.

Step 2: If the underlined contains a verb, check whether the subject and verb agree both in
number and person. Also the tense of the verb should agree with the time sequence.

Step 3: If the underlined contains pronouns, the pronouns must agree with their antecedents in
both number and person. If it contains several pronouns, the pronouns should refer clearly to their
antecedents.

Step 4: If the underlined contains a word of comparison, make sure the comparison is logical,
clear and paralleled.

Step 5: If the sentence contains a modifier of phrase, check whether it modifies the subject
clearly.

Step 6: If the underlined contains a clause, make sure the introductory pronouns (where, which)
is correct. If the sentence contains a subjunctive, is that omitted? verb in present tense?

Step 7: If the underlined contains the conjunction or, and, check if similar elements in a list should
be in similar form.

Step 8: If the underlined contains an idiom, check if the idiom is correct and complete.

Step 9: If the underlined conforms to all the above grammar rules, check if the sentence is clear,
concise, and without awkward.

79
Chapter 3 Critical Reasoning

On the test day, you will encounter 41 verbal questions, 10 out of them are Critical Reasoning. To
make it comfortable for you to prepare for this subtest, we have designed an interesting "number"
course (the name of each strategy begins with a number) to help you learn the test prep
strategies.

Section 1: Introduction

One Definition

Four Elements of an Argument

Seven Common Fallacies

Three-element Rule

Two Traps

Five Answer Choices

Section 2: Six Types of Reasoning

Section 3: Eight Types of Question

80
Section 1: Introduction to Critical Reasoning

1. One Definition: Argument

Most people call the Critical Reasoning as Argument. An argument is a coherent series of
statements leading from a premise to a conclusion. Formula of argument looks something like this:

Premise + Example/Reason = Conclusion

Depending on the effectiveness of premises and the logicality of reasoning (use of example and
reason), an argument can be perfectly true or totally fallacious. For instance, someone may
conclude Candidate G will become the next president of United States because in a recent poll,
eight out of ten gave votes to him. This poll can be effective if the respondents are representative
to the whole electorate, or be ineffective if the respondents are biased sample.

However, the critical reasoning question never asks you to determine whether an argument is
correct or incorrect. Rather, the question requires you to evaluate the reasoning of an argument. In
other word, arguments are designed to test your ability to think logically, not your ability to seek
truth.

Lets look at a sample argument:

Our work proves to be very successful. In the past three years, each of our five clients has
experienced the fastest growth of sales in their history. Therefore, if your company meets
management problems, do not hesitate to call Sigma & Max, since we are the best management
consulting company.

Here, the advertiser tried to convince that it is the best management consulting company available
and to persuade its potential customer to choose its service. To support its statement, the
advertiser cited five of its clients, each of them gained the fastest growth rate in their history.

81
2. Four elements of an argument

An argument is a coherent series of statements leading from a premise to a conclusion. One can
cite evidences to strengthen an argument or attack its assumption to weaken an argument.

Premises + Evidence = Conclusion

I. Conclusion

How do you identify the authors conclusion? Most often, a conclusion is stated in the last or first
sentence in an argument. The conclusion rarely comes in the middle of an argument. Also, you
can search for the conclusion indicators that are commonly used to introduce a conclusion.

Conclusion Indicators

so thus therefore as a result

consequently accordingly hence imply

conclude that follows that means that infer that

II. Premise

Premise is the fact or reason that the author uses to strengthen his argument. The
following are phrases that introduce the premises of an argument.

Premise Indicators

because since for as

if assume suppose evidence

the reason is may be derived


on the basis of in that
that from

III. Counter-evidence

Sometimes, the author uses counter-evidence words to argue against his opposite, or concede
certain minor points that may weaken his argument. For the latter, the counter evidence is finally
refuted by further evidence. Following are some of the most common used counter-evidence
indicators.

82
Counter-evidence Indicators

actually despite admittedly except

even though nonetheless nevertheless although

however In spite of do may

IV. Assumptions

Assumptions are those that the author uses to strengthen his argument but leaves it unstated.

The assumption is a gap between the premises and the conclusion. In order to evaluation an
argument, you always have to find this gap.

83
3. Seven Common Fallacies

There are numerous fallacies that you may encounter on the test day. To help you easily identify
the logical flaws on the test day, we have summarized the three most common logical fallacies.

I. Biased Sample Fallacy

The biased sample fallacy is committed when the author draws a conclusion concerning a
population based on a biased sample. Here is an argument that commits the fallacy of the biased
sample:

In a recent survey conducted by Computer Weekly, 93% of the respondents indicated that
Computer Weekly is the best computing magazine that they had subscribed. This survey clearly
shows that Computer Weekly is the best computing journal available.

The data of 93% are drawn from a sample that does not represent the entire electorate. Since the
survey was conducted by people who had subscribed Computer Weekly, not all subscribers of
computing journals have an equal chance of being included in the sample. In other words, people
who had read Computer Weekly might be more likely to have a favorable opinion on Computer
Weekly. Therefore, the sample is not representative, and is biased.

II. Insufficient Sample Fallacy

When someone draws a conclusion about a population based on a sample that is not large
enough, he or she commits the insufficient sample fallacy. A typical form would be:

1) X% of A are B.

2) Therefore, A are B.

III. Questionable Cause Fallacy

This fallacy has the following general form:

1) A happens before B.

2) Thus A is the cause of B.

It is an error in reasoning to conclude that one thing causes another simply because one happens
before the other. The fallacy committed is that a causal conclusion is being drawn from
inadequate evidence.

IV. Personal Attack

84
Personal attack, as its name suggests, is a technique to challenge a persons character instead of
his opinions. As a matter of fact, there is no correlation between personal character and his
position. A sample statement would be:

Dont believe him; he had a bad credit.

V. Equivocation

Equivocation is the use of a word in two or more meanings in an argument. The author tries to
prove his statement, but the underlying subject is in a second meaning other than the most
commonly used.

The average normal infant born in the United States weighs between twelve and fourteen pounds
at the age of three months. Therefore, if a three-month-old child weighs only ten pounds, its weight
gain has been below the United States average.

Here, the flaw is that average weight gain is not the same as average weight.

VI. Appeal to authority

Someone is appealing to authority when he cites an experts opinion to support his own position. It
is obvious that an experts statement can not be always true. Especially if the expert is not
specializing in the filed related to the argument, his opinion is very likely to be fallacious. Here is
an example.

Mr. Kate can definitely solve the management problems of our company because Mr. Kate is a
professor in Management.

Its true that Mr. Kate is an expert in management research, but not necessarily in real-world
management experience.

VII. Faulty Analogy

Faulty analogy is committed when someone reasons that two things are similar in this way
because they are similar in that way. The following argument commits the faulty analogy.

Peter and Jennifer were twins. Since Peter like dancing, Jennifer will also like dancing.

85
4. Three-element Rule

Known as "Transitive Property" or "Syllogism", the three-element rule is among the most common
reasoning methods you can use to solve GMAT Critical Reasoning problems. This rule could be

expressed by the formula: If A B and B C , then, A C .

Example
Bill earns more commission than does Sandra. But since Andrew earns more commission than
does Lisa, it follows that Bill earns more commission than does Lisa.
Any of the following, if introduced into the argument as an additional premise, makes the argument
above logically correct EXCEPT:
A. Andrew earns more commission than Bill
B. Sandra earns more commission than Lisa
C. Sandra earns more commission than Andrew
D. Sandra and Andrew earn the same amount of commission
E. Bill and Andrew earn the same amount of commission

The question asks for an additional premise that does NOT make the argument logically correct.
Adding A to the information given in the passage leaves open the possibility that, in order of
commission, the people rank: Andrew, Lisa, Bill, Sandra. Because this order is contrary to the
conclusion of the argument, A leaves open the possibility that the conclusion of the argument is
false; it is thus the best answer. By contrast, any of other choices, when added to the information
that the commission Bill earns is greater than that of Sandra and that the commission Andrew
earns is greater than that of Lisa, makes the conclusion-that Bill earns more commission than
Lisa-follow logically.

86
5. Two Traps

Opposites and scope traps are the two most commonly used traps that the test writers use to fool
the unskilled test takers.

Opposites Trap

Here are examples:

1. "Which of the following mostly weakens the above argument?" The test writers present answer
that actually strengthens the argument. The earlier the answer appears (say choice A), the more
likely it is that you will pick up this answer.

2. The question asks for an assumption for an argument but presents an answer choice that
summarizes the argument. Again, the earlier the answer appears (say choice A), the more likely it
is that you will pick up this answer.

3. The question asks for an answer that can not strengthen/weaken an argument but provides a
choice that can be. For example, Which of the following can not be a possible explanation?

The Scope Trap

Be alert to scope words that are too narrow or broad. These extreme words are often included in a
stuff choice. The answer choice that uses moderate tone (such as usually, sometimes or
generally) is generally safe and therefore the correct answer.

Extreme Quantifiers

all except only never

must always no everywhere

Moderate Quantifiers

probably generally likely some

most many sometimes may

87
6. Five Answer Choices

On the test day, each of the five choices will follow into one of the categories below.

(1) Correct answer choice. The best answer is the correct answer. As we said at the beginning of
this chapter, it is not necessarily the truth. Be alert to what the question asks.

(2) Opposite answer choice. The opposite answer is exactly opposite to the correct answer. The
test writer likes it very much because it takes minutes to create but fool most test-takers.

(3) Biased answer choice. The biased choices are also the common trick. Many unskilled
test-takers often pick up this acceptable one. The earlier the choice appears, the more students
are fooled.

(4) Irrelevant answer choice. This choice talks about something that is not related to the passage
or the questions. It is easy to eliminate such choice.

(5) Biased or Irrelevant answer choice. The fifth answer can be either biased or irrelevant.

Example

The average net profit margin for the U.S. companies as a whole is 4.5 percent, but
companies located in Washing will achieve an average net profit margin of 7.8 percent, and
those located in Utah, 3.2 percent. If a firm in Utah were to move to Washing, therefore its net
profit margin would be expected to achieve higher profit margin than would be the case if it
remained in Utah.

Which of the following, if true, would most seriously weaken the conclusion drawn in the passage?

A. Consultants in management consulting companies do believe that moving to Washington will


significantly increase the average net profit margin.

B. The governor of Utah has falsely alleged that statistics for his state are inaccurate.

C. The higher net profit margin ascribed to Washingtons current companies is attributable mostly
to the small software companies that contributed to over 80 percent of profits generated by all
Washingtons companies.

D. Thirty percent of all Utahs companies can expect to achieve 7.8 percent of net profit margin.

E. Washingtons companies pay tax well below the national average for the United States.

Choice A actually strengthens the conclusion.

88
For choice B, because the governor's allegation is false, it cannot affect the conclusion.

Choice C suggests that a significant proportion of Washington companies profits are generated by
small software companies. Since Utahs companies do not necessarily do business in software
industry, this choice presents a reason to doubt that non-software companies will achieve an
increased margin. Therefore, this choice is the best answer.

Choice D fails to weaken the conclusion because it is consistent with the information given and the
conclusion about net profit margin.

By suggesting that Washingtons companies pay tax well below the national average for the
United States, choice E supports the conclusion.

89
Section 2: Six Types of Argument

There are major two categories of argument, deductive and inductive argument. An argument is
deductive if its conclusion necessarily follows from its premise. The most commonly used
deductive arguments in GMAT are Mathematical Reasoning and If-Then Theory. Unlike that of the
deductive argument, the conclusion of an inductive argument can be highly likely or highly unlikely
depending on how convincing the premises are. Typical inductive arguments include Generation,
Analogy, and Casual Reasoning. In the following passages, we will introduce them one by one.

1. Deductive Argument

A. Statistical Reasoning
To reason statistically is to make a conclusion by conducting an arithmetical calculating. For
instance, if x = 1and y = 3, then x < y. Of course, this reasoning has its own premise. But this
premise is basically indubitable: 1 < 3. In evaluating an argument that contains mathematical
calculation, you should try to identify the underlying comparison between two subjects. There are
three major concepts that would help you evaluate the validity of a particular argument:
comparative value, ratio, and statistical sample.

Concept 1 Comparability: comparative and absolute value


There is a big difference between comparative value and absolute value. The former is a ratio,
while the later is a value.

Comparison of absolute value:


A1 B1

A1 B1

Comparison of comparative value:
Atotal Btotal

The following reasoning is ridiculous because it mistakes absolute value as comparative.

During the SARS days, about 23,500 doctors who had treated SARS sufferers died and about
23,670 doctors who had not engaged in treatment for SARS sufferers died. On the basis of
those figures, it can be concluded that it was not much more dangerous to participate in
SARS treatment during the SARS day than it was not to participate in SARS treatment.

Which of the following would reveal most clearly the absurdity of the conclusion drawn above?
A. Counting deaths among doctors who had participated in SARS treatment in addition to
deaths among doctors who had not participated in SARS treatment

90
B. Expressing the difference between the numbers of deaths among doctors who had treated
SARS sufferers and doctors who had not treated SARS suffers as a percentage of the total
number of deaths
C. Separating deaths caused by accidents during the treatment to SARS suffers from deaths
caused by infect of SARS suffers.
D. Comparing death rates per thousand members of each group rather than comparing total
numbers of deaths
E. Comparing deaths caused by accidents in the United States to deaths caused by infect in
treating SARS suffers.

Concluding from the similar numbers of deaths in two groups that the relative danger of death was
similar for both groups is absurd if, as here, one group was far smaller. D exposes this absurdity
by pointing out the need to compare death rates of the two groups, which would reveal the higher
death rate for the smaller group. Therefore, D is the best answer. Since the conclusion
acknowledges the difference between the number of deaths of doctors who treated SARS suffers
and doctors who had not treated the SARS suffers, expressing this difference as a percentage, as
suggested by B, is beside the point. A is inappropriate because it simply adds a third group to the
two being compared. Because cause of death in not at issue, C and E are irrelevant.

Concept 2 Ratio
This kind of reasoning is similar to the McKinsey's 20-80% rule (20% of our clients generates 80%
of total profits). Let's look as an example to see how this rule can help us solve a reasoning
problem.

A proposed ordinance requires the installation in new taxis of alarm system automatically
triggered by the presence of a highjack. However, a taxi driver argued that because more
than ninety percent of highjack is alarmed by taxi drive, alarm system would only marginally
decrease accidents caused by highjack.

Which of the following, if true, would most seriously weaken the taxis drivers argument?
A. most taxi drivers have no formal training in how to alarm a highjack.
B. Since new taxis are only a tiny percentage of available taxi in the city, the new ordinance
would be extremely narrow in scope.
C. The installation of radar in new taxis costs significantly less than the installation of alarm
system.
D. In the city where the ordinance was proposed, the average time required by the police
department to respond to a highjack was less than the national average.
E. The largest proportion of damage that results from highjack is caused by highjack that starts
when taxi drivers fails to alarm.

91
The taxi driver reasons from evidence about most highjack to a conclusion about the effectiveness
of alarm system in preventing damage. But this reasoning is faulty because of the possibility that
most of the damage results from the minority of highjack excluded from the taxi driver's evidence.
That possibility is realized if E is true. Thus, E is the best answer. Because the taxi driver's
argument concerns neither the cost of installing alarm systems not a comparison with police
department performance in other locations, C and D are irrelevant. The evidence the taxi driver
cites suggests that formal training is not needed in order to alarm highjack. So A is not the correct
answer. B supports the taxi driver's view that requiring alarm system would have a limited effect.

Concept 3 The whole-and-individual

The general statistical data does not necessarily apply to an individual. For instance, as a whole,
United States is wealthy than China. But we can not say every U.S. resident is wealthier than that
in China. To better understand how the whole concept does not apply to an individual, let's look at
the following example.

Surveys show that in the next 3 years, the population of old age will increase by 20 percent. In
order to take this advantage, the department store should replace 20 percent of its goods for
young age consumers with goods for old age consumers.

Of the following, the best criticism of the conclusion that inducing department store owner to
replace goods is that the conclusion is based on

A. the fact that the old-age population will increase by less than 20 percent in the town where the
department store is located.
B. past patterns of consuming and may not carry over to the future
C. the assumption that each old consumer is loyal to a single department store at any one time
D. the assumption that each department store sells goods for both young and old people
E. figures for the retail industry as a whole and may not hold for a particular department store

Here, "the population of old age will increase by 20 percent" is a statistical data. But the conclusion
is based on figures for the retail industry as a whole and may not hold for a particular department
store as demonstrated by the choice of E.

92
B. If-Then Theory
If the statement "if A, then B" is true, then its contra-positive "if not B, then not A" is also true. We

can diagram this reasoning by followings diagram. If A B , then ~ B ~ A .

Example

In 2003 an airline in United State lost more than half, on average, of the foreign passengers
they had previously served each year. Researchers have alleged that this extreme drop
resulted from a rise in price of tickets for international lines from $60 to $90 per 1,000 miles.

Which of the following, if feasible, offers the best prospects for alleviating the problem of the drop
in passengers as the researchers assessed it?
A. Cooperating with other airlines to provide more international lines.
B. Allowing foreign passengers to pay the same as the previous international line
C. Reemphasizing the goals and mission of the airline as serving both domestic passengers and
foreign passengers
D. Increasing the financial resources of the airline by raising the ticket price for domestic
passengers
E. Offering superior VIP service for foreign passengers.

The researchers attribute the drop in passengers of foreign passengers to an increased price in
ticket. If researchers are correct, reducing these prices should halt the drop in passenger. B offers
a plan for reducing these prices and so is the best answer. The reasoning of the stimulus can be
expressed as:

A (a rise in price of ticket for foreign passengers)=>B (extreme drop of passengers )

Then ~B (drop of passengers did not continue)=>~A (price decreases for foreign passengers)

None of C, D and E offers a plan that would reduce the prices taken to be responsible for the drop
in passengers. Nor does A offer such a plan: because the problem to be addressed is a drop in
foreign passengers, providing more international lines, as A suggests, would offer no prospect of
alleviating the problem.

93
2. Generalization

Generalization is an argument that draws a conclusion about a population based on a sample. If


the sample is too small or unrepresentative, the conclusion can be fallacious. If the sample is
representative or additional sample is added, then the conclusion can be solid.

Example

Toughened recruiting criterions have not been the primary cause of the present enrollment
drop in a certain business school. The drop of enrollment is primarily caused by the fact that
in recent years Career Office have not experienced any improvements in assisting graduates
to seek jobs.

Which of the following, if true, would most support the claims above?
A. Many graduates already in the job market would not have been admitted under the new
recruiting standards.
B. Today more students are entering the school with a higher credential than in the past.
C. Some students have cited higher standards for recruiting as a reason for the current enrollment
drop.
D. Many applicants have cited low quality career help as reasons for not applying to the school.
E. Many prospective students have cited the new recruiting standards as a reason for not entering
the school.

The passage rejects one explanation of the enrollment drop of students-that it results from
toughened recruiting standards-and advances an alternative-that it results from deficiencies in
career assisting. D provides corroborative evidence for the latter explanation by suggesting that,
for applicants, poor career service were reasons for their decision of not applying the school.
Therefore, D is the best answer. Choices A, C and E provide evidence that tends to implicate new
recruiting standards in the enrollment drop, and thus support the explanation that the passage
rejects. B describes what may be a result of the new recruiting standards, but it provides no
evidence favoring one explanation of the enrollment drop over the other.

94
3. Analogy

Analogy is an argument that two things are similar in this way because they are similar in that way.
The greater the similarity between the two things being compared, the stronger the argument will
be.

Let's look at an example.

Example

Contrary to the argument made by its opponents, the down-sizing plans are justified.
Opponents should remember that the Centralization pulled this company out of great
economic troubles even though some of its programs were later found to be unconstitutional.

The opponents could effectively defend their position against the author's strategy by pointing out
that
A. the expertise of those opposing the plan is outstanding
B. the lack of justification for the new down-sizing plan does not imply that those who drew it up
were either inept or immoral
C. the practical application of the new down-sizing plan will not entail indiscriminate employees
cuts
D. economic troubles present at the time of the Centralization were equal in severity to those
that have led to the present plan
E. the fact that certain flawed programs or plans have improved the economy does not prove
that every such program can do so

To attack the author's conclusion, the opponents could point out that even if new down-sizing plan
and Centralization are similar in some respects, the other respects of these two programs will not
be necessarily the same.

95
4. Causal Reasoning

Confusing Correlation with Causation is the most common fallacy associated with causal
reasoning. It is clearly questionable to conclude that one thing causes another simply because
one happens before the other. It may be only coincidental that they occurred together, or
something else may have caused them to occur together. However, if additional evidence is
provided, the conclusion can be compelling.

Example

Our work proves to be very successful. In the past three years, each of our five clients has
experienced the fastest growth of sales in their history. Therefore, if your company wants to
increase sales, do not hesitate to call Sigma & Max, since we are the solution.

Which of the following, if true, most seriously jeopardizes the validity of the argument by the
speaker above?
A. Most of the consultants at Sigma & Max hold MBA degrees.
B. Even without the help of Sigma & Max, the five clients of Sigma & Max will achieve the same
growth rate in sale.
C. Sigma & Max is one of the five leading management consulting companies.
D. Sigma & Max uses an updated accounting approach to help companies to cut cost.
E. All of the five clients of Sigma & Max are doing business in financial industry.

As B states, without the help of Sigma & Max, the five clients of Sigma & Max will achieve the
same growth rate in sale. Therefore, B is the best answer. Choices C and D exactly strengthen the
argument by the speaker. The first and last choices are irrelevant to the evaluation of the
argument above.

96
5. Finding Assumption

While the above four types of logic help you evaluate the effectiveness of an argument in logical
thinking, this method assists you in weakening or strengthening an argument by attacking its
assumption. As I said at the beginning of this chapter, a typical inductive argument is formulated
as:

Premises + Assumptions = Conclusion

The best way to weaken an argument is to attack its assumption. Let's look at an example:

Milk is a perfect food because it provides protein and calcium.

The assumption underlying for this reasoning is:

Any food that provides protein and calcium is a perfect food.

If you can name a food which provides protein and calcium, yet is not a perfect food, then you
automatically weaken this argument.

Example #1

If highways were restricted to cars and only those truck with capacity of less than 8 tons, most
the truck traffic would be forced to run outside highway. Such a reduction in the amount of
truck traffic would reduce the risk of collision in highway.

The conclusion draw in the first sentence depends on which of the following assumptions?

97
A. The roads outside highway would be as convenient as highway for most drivers of truck.
B. Most roads outside highways are not ready to handle truck traffic.
C. Most trucks that are currently running in highway have a capacity of more than 8 tons.
D. Cars are at greater risk of becoming involved in collisions than are trucks.
E. A reduction in the risk of collision would eventually lead to increases in car traffic.

The first sentence concludes that prohibiting trucks with capacity of more than 8 tons from highway
would force most trucks away from highways. This conclusion cannot be true unless it is true that,
as C says, most trucks that use highways have capacity of more than 8 tons. Therefore, the first
sentence's conclusion assumes this choice, which is thus the best answer. The conclusion need
not assume that roads outside highways are convenient for trucks (A), since the restrictions would
give trucks that have a capacity of more than 8 tons no choice. The conclusion concerns only how
the restriction would affect the volume of truck traffic, so B, D and E, which deal with cars and with
risk of collision, need not be assumed.

Example #2
2. Which of the following, if true, would most strengthen the conclusion drawn in the second
sentence?
A. Cars with a capacity of more than 8 tons are already excluded outside highways.
B. Highways are experiencing overcrowded traffic primarily because of sharp increases in car
traffic.
C. Many drivers of trucks would rather buy truck with a capacity of less than 8 tons than be
excluded from highways.
D. The number of collisions that occur near highways has decreased in recent years.
E. Trucks that have a capacity of more than 8 tons cause a disproportionately large number of
collisions in highways.

The second sentence concludes that the reduction described in the first sentence would reduce
the risk of collisions in highways. According to E, such a reduction would remove precisely the kind
of truck that causes a disproportionate number of collisions. Thus, E is the best answer. Because
A does not address the question of whether reducing car traffic would reduce the risk of collisions,
it is inappropriate. B and C concern the question of whether or not the proposed restrictions would
reduce highway traffic, but not the question of whether any resulting reductions would reduce the
risk of collisions. That the number of collisions has recently decreased is irrelevant to whether the
proposed reduction would further reduce collisions, so D is inappropriate.

98
6. Business Thinking

The test-maker always says that GMAT does not test specific knowledge in business. But as a
matter of fact, over one-thirds of GMAT questions are in business subject. In critical reasoning, the
percentage is as high as 50%. If you do not possess any business background or knowledge, you
will find many questions difficult to answer. Be sensitive to basic business knowledge will definitely
help you evaluate an argument.

Let's look at an example:

Example

A life insurance company allows people to prepay their endowment insurance at current rates.
The policyholder then pays the premium every year. People should participate in the program
as a means of decreasing the cost for their living after retirement.

Which of the following, if true, is the most appropriate reason for people NOT to participate in the
program?
A. Peoples are unsure about which insurance company they will choose after retirement.
B. The amount of money accumulated by putting the prepayment funds in an interest-bearing
account today will be greater than the total cost of insurance when they retire.
C. The annual cost of premium is expected to increase at a faster rate than the annual increase in
the cost of living.
D. Some of the insurance companies are contemplating large increases in premium next year.
E. The prepayment plan would not cover the cost of hospitalization.

The passage recommends that people participate in a premium prepayment program as a means
of decreasing the living cost after they retire. If B is true, placing the funds in an interest bearing
account would be more cost-effective than participating in the prepayment program. Therefore, B
would be a reason for NOT participating and is the best answer. A is not clearly relevant to
deciding whether to participate since the program applies to whatever insurance companies they
choose. C and D, by stating that premium will increase, provide support for participating in the
program. E is not clearly relevant to deciding whether to participate, since the expenses
mentioned fall outside the scope of the program.

99
Section 3: Eight Types of Question

There are generally eight types of question you expect to encounter on your test day. Among them,
Boldface Question is the new one in CAT. You should be familiar with the following question types.

1. Inference Question

Inference Question, also known as Conclusion Question, asks you to draw a conclusion based on
the stimulus. The following are the typical Inference Questions:

Based on the statements above, which of the following can be properly drawn?

Which of the following can be implied in the passage above?

If the statements above are true, which of the following must also be true?

Which of the following inferences is best supported by the statements above?

How to solve Inference Question:

1. Read the statement and look for a comparison. The stimulus always compares two things,
events, or time sequences.

2. Try to figure out what causes the difference before you look at the answer choices.

3. Use POE to eliminate those choices with extreme quantifiers, such as all, never, must.

Example

The price of purchasing a car in Country Q is 120 percent less than the price of purchasing a car in
Country Y. Even after transportation fees and tariff charges are added, it is still cheaper for a buyer
to import car from Country Q to Country Y than to buy car in Country Y.
The statements above, if true, best support which of the following assertions?
A. Gasoline prices in Country Q are 120 percent below those in Country Y.
B. Importing cars from Country Q to Country Y will eliminate 120 percent of the sales of cars in
Country Y.
C. The tariff on a car imported from Country Q to Country Y is less than 120 percent of the price of
a car in Country Y.
D. The fee for transporting a car from Country Q to Country Y is more than 120 percent of the price
of a car in Country Q.
E. It takes 120 percent less time to transport a car in Country Q than it does in Country Y.

If the tariff on importing cars from Country Q to Country Y were as high as 120 percent or more of
the price of purchasing a car in Y, then, contrary to what the passage says, the price of importing

100
cars from Q to Y would be equal to or more than the price of a car in Y. thus, the tariff cannot be
that high, and C is the best answer. A and E give possible partial explanations for the price
difference, but neither is supported by the passage because the price advantage in Q might be
attributable to other factors. B and D are both consistent with the information in the passage, but
the passage provides no evidence to support them.

101
2. Assumption Question

The Assumption Question requires you to identify the assumption in an argument. An assumption
is a premise that a conclusion depends on but leaves it unstated. Typical Assumption Question
looks like this:

The official's conclusion logically depends on which of the following assumptions?


The argument above assumes that.

How to solve Assumption Question

1. Read the argument and find its premise and conclusion.

2. Consider what is needed to make the conclusion compelling.

3. Review the answer choices and identify the gap that bridges the premise and conclusion.

Example

To protect our business secrets, our company could prohibit high-level executives from accepting
similar positions in our competitors for three years after such executives leave our company. One
such manager concluded, however, that such a prohibition would be unfortunate because it would
prevent high-level executives from earning a livelihood for three years.
The executive's conclusion logically depends on which of the following assumptions?
A. Laws should not restrict the behavior of former company executives.
B. Company executives are typically people who have previously been high-level executives from
its competitors.
C. Low-level company executives do not often seek similar position when they leave company.
D. High-level company executives who leave company are capable of earning a livelihood only as
lobbyists.
E. High-level company executives who leave company are currently permitted to act as lobbyists
for only three years.

The executive argues that prohibiting high-level company executives from accepting similar
positions at its competitors for three years would prevent the executives from earning a livelihood
for that period. The reasoning tacitly excludes the possibility of such executives earning a living
through work other than in the same industry. Therefore, D, which expresses this tacit assumption,
is the best answer. The executive's argument does not depend on the assumption in A, since the
argument would not be invalidated if some restrictions on the behavior of company executives
were desirable. The executive's argument does not depend on the assumption in B, since the
argument would not be invalidated if executives were not typically former high-level company
executives of its competitors. The executive's argument does not depend on the assumption in C,

102
since the argument would not be invalidated if former low-level company executives did often
become lobbyists. The executive's argument does not depend on the assumption in E, since the
argument would not be invalidated if former high-level company executives could act as high-level
executive in its competitors indefinitely.

103
3. Strengthen Question

Assumptions connect premises to conclusions. You may strengthen an argument by strengthening


the assumptions. Here are some examples of Strengthen question stems:

Strengthening:

Which of the following statements, if true, would most significantly strengthen the conclusion
drawn in the passage?
Which of the following, if true, would strengthen the argument presented above?

How to solve Strengthen Questions

1. Read the argument and find its premise and conclusion.

2. Look for assumptions.

3. Read the questions. Use POE until you there is only one choice.

Example

The average net profit margin for the U.S. companies as a whole is 4.5 percents, but
companies located in Washing will achieve an average net profit margin of 7.8 percents, and
those located in Utah, 3.2 percents. If a firm in Utah were to move to Washing, therefore its
net profit margin would be expected to achieve higher profit margin than would be the case if
it remained in Utah.

Which of the following statements, if true, would most significantly strengthen the conclusion
drawn in the passage?

A. As company density increases in Washington, margin figures for that state are likely to be
revised downward.

B. Market demand factors tending to favor profit margin are abundant in Washington and less
numerous in Utah.

C. Twenty-five percent of all Utahs companies that move to Washington achieve margin of more
than 7 percents.

D. Over the last decade, net profit margin has risen at a higher rate for Utahs companies than for
Washingtons companies.

E. Studies show that the average net profit margin for Utahs companies that move permanently to
Washington is roughly equal to that of Washingtons companies that remain in Washington.

104
If B is true, the greater abundance of margin-promoting market demand factors it mentions is
probably at least partly responsible for the higher margin in Washington. Therefore, B is the best
answer.

105
4. Weaken Question

Assumptions connect premises to conclusions. You may weaken an argument by weakening the
assumptions. Here are some examples of Weaken question stems:

Weakening:

Which of the following statements, if true, would most significantly weaken the conclusion drawn
in the passage?
Which of the following, if true, would weaken the argument presented above?

How to solve Weaken Question

1. Read the argument and find its premise and conclusion.

2. Look for assumptions.

3. Read the questions. Use POE until you there is only one choice.

Example

The average net profit margin for the U.S. companies as a whole is 4.5 percent, but
companies located in Washing will achieve an average net profit margin of 7.8 percent, and
those located in Utah, 3.2 percent. If a firm in Utah were to move to Washing, therefore its net
profit margin would be expected to achieve higher profit margin than would be the case if it
remained in Utah.

Which of the following, if true, would most seriously weaken the conclusion drawn in the passage?

A. Consultants in management consulting companies do believe that moving to Washington will


significantly increase the average net profit margin.

B. The governor of Utah has falsely alleged that statistics for his state are inaccurate.

C. The higher net profit margin ascribed to Washingtons current companies is attributable mostly
to the small software companies that contributed to over 80 percent of profits generated by all
Washingtons companies.

D. Thirty percent of all Utahs companies can expect to achieve 7.8 percent of net profit margin.

E. Washingtons companies pay tax well below the national average for the United States.

Choice A actually strengthens the conclusion.

For choice B, because the governor's allegation is false, it cannot affect the conclusion.

106
Choice C suggests that a significant proportion of Washington companies profits are generated by
small software companies. Since Utahs companies do not necessarily do business in software
industry, this choice presents a reason to doubt that non-software companies will achieve an
increased margin. Therefore, this choice is the best answer.

Choice D fails to weaken the conclusion because it is consistent with the information given and the
conclusion about net profit margin.

By suggesting that Washingtons companies pay tax well below the national average for the
United States, choice E supports the conclusion.

107
5. Paradox Question

The Paradox Question presents you with a seeming paradox situation in an argument, and then
asks you to seek an explanation on how that discrepancy can exist. Typical ways in which these
questions are asked include:

Which of the following statements, if true, would best explain the paradox described above?

Which of the following can explain the apparent contradiction above?

How to solve paradox question

1. Read the argument and identify the apparent contradiction.

2. Look for the sequential difference or spatial difference since they may be the reason that causes
the seeming paradox.

3. If you are unable to pick up the correct answer by step 2, pay attention to the subjects in the two
contradictive situation. Are the two subjects the same? If not, the difference could be the reason for
paradox.

Example

In 1992, 5 percent of every dollar paid in tax went to support the unemployed citizens. In 1998,
8 percent of every dollar paid in tax went to such funds, although that unemployment rate has
decreased in 1998 than in 1992.

Each of the following, if true, could explain the simultaneous increase in percent of every dollar
paid in tax to support the unemployed citizens and decrease in the number of unemployment rate
EXCEPT:

A. On average, each unemployed citizen received more money in 1998 than 1992.

B. On average, people paid less tax in 1998 than in 1992.

C. The individuals had paid more tax than did enterprises during this period.

D. Income before tax has significantly decreased since 1992.

E. The number of tax evaders rose sharply between 1992 and 1998.

Choice A suggests that the total amount of dollars used to support unemployment has increase,
therefore explain the paradox. Choice B, D, and E all suggests that the amount of tax collected
decreased, thus percent of every dollar that went to support the unemployment increases. Only
choice C does not explain such paradox, therefore is the correct answer.

108
6. Reasoning Question

Reasoning questions ask you to describe how the line of reasoning of an argument. Here are
some examples:

The author's point is made by.


A attacks Bs position by.
How does the author make his point?

How to solve Reasoning Question

1. Read the argument and identify its premise and conclusion.

2. Identify how the premises and/or evidences lead to the conclusion.

3. Pick up the correct answer based on your awareness of typical reasoning methods. The most
commonly used reasoning ways are Analogy, Generalization, Causal Reasoning and Attacking
Assumption.

Example

Contrary to the argument made by its opponents, the down-sizing plans are justified.
Opponents should remember that the Centralization pulled this company out of great
economic troubles even though some of its programs were later found to be unconstitutional.

The author's method of attacking the opponents argument is to


A. attack the character of the opponents rather than their claim
B. imply an analogy between the current down-sizing plan and a previous Centralization
program
C. point out that the opponents' claims imply a dilemma
D. show that the opponents' reasoning leads to an absurd conclusion
E. show that the Centralization also called for down size the employee pool

The argument of the stimulus goes like this: the new down-sizing plan and Centralization are
similar to each other because both are improper. Since the Centralization pulled this company out
of great economic troubles, therefore, the new plans of down size are justified. Obviously, the
speaker used analogy to make a conclusion.

109
7. Complete Question

As its name suggests, this type of question asks you to complete a reasoning that contains a blank.
This type is seldom in GMAT critical reasoning. However, if you do encounter it on the test day,
this passage will help you quickly identify the right answer choice.

The blank needed to complete is always a part of the reasoning presented in the stimulus. That is
to say, one of the following five choices, will bridge the premise and conclusion. So, what you need
to do is to find a statement or fact that would make the above argument complete, reasonable and
persuasive.

How to approach Complete Question

1. Look for gaps between the premises and the conclusion. Ask yourself why the conclusion is
true. Before you progress to the answer choices, try to get feel of what statement is necessary
to fill that gap between the premises.

2. Eliminate the choices that can't make the argument reasonable.

Example

Which of the following best completes the passage below?

In a survey of law school applicants, two-fifths admitted to create at least one reference letter
by themselves. However, the survey may underestimate the proportion of law school
applicants who create reference letter by themselves, because____.

A. some applicants who create reference letter by themselves taking the survey might have
claimed on the survey not to create reference letter by themselves
B. some law school applicants who create reference letter by themselves taking the survey might
have claimed on the survey to create reference letter by themselves
C. some law school applicants who claimed on the survey to create at least one reference letter
may create more than one reference letter
D. some law school applicants who claimed on the survey to create reference letter by themselves
may have been answering exactly
E. some applicants who are not applying for law school probably create at least one reference
letter

A is the best answer. If law school applicants who in fact create reference letter by themselves
claimed not to create reference letter by themselves, the survey results would show a smaller
proportion of law school applicants who create reference letter by themselves than actually exists.

110
Therefore, this choice is the best answer. B is inappropriate because generally law school
applicants who do not create reference letter by themselves claimed to create reference letter by
themselves could contribute to the overestimation, but not to the underestimation, of applicants
who create reference letter by themselves. D is inappropriate because applicants who admitted
that they create reference letter by themselves would not contribute to an underestimation of the
proportion of applicants who create reference letter by themselves. C and E are inappropriate
because the argument is concerned neither with the number of reference letters created by
applicants nor with the applicants to non-law school.

111
8. Boldface Question

In this type, one or more sentences in the stimulus are in boldface. The question following requires
you to identify the logical relationship between the boldfaced sentences, or how it relates to a
particular position (the author agree or disagree).

Boldface question is totally new for computer-based test, but is easy to tackle. All you have to do is
to understand the argument: identify the conclusion, evidence, and the reasoning from evidence to
conclusion. Sometime, you are required to critique the validity of the argument.

How to approach Boldface Question

1. Identify the conclusion. Ask yourself what the conclusion is, what the author trying to prove, or
what the authors main point is.

2. Look for the evidence that the author uses to support or argue against a position.

3. Search for argument indicator to determine the relationship between evidence and
conclusion.

Conclusion Indicators

so thus therefore as a result

consequently accordingly hence imply

conclude that follows that means that infer that

Premise Indicators

because since for as

if assume suppose evidence

on the basis of the reason is that may be derived from in that

Counter-evidence Indicators

actually despite admittedly except

even though nonetheless nevertheless although

however In spite of do may

112
4. Before you pick up the answer choices, make sure you did NOT mistake counter-evidence as
evidence.

Let's look at a relatively easy question.

Example #1

To be eligible for a U.S. student visa, an applicant must have a valid passport, and have been
accepted by a U.S. institution. Peter, a French applicant, has a valid passport and been
admitted to Western California State College, so he must be issued a U.S. student visa.

The two portions in boldface play which of the following roles?

A. The first is the part of evidence in support of this argument; the second is the conclusion that
could not be drawn from all evidence that the argument contains.

B. The first is the first-evidence that supports this argument; the second is the main point that must
be drawn from all evidence that the argument includes.

C. The first is the one fact of two that argument includes; the second is the conclusion that could
be drawn from this passage.

D. The first is the background that is necessary for this argument; the second is the conclusion that
is not drawn only from the first.

E. The first is the cause that the argument includes; the second is the effect that can be drawn only
from this cause.

In this argument, the author first introduced the requirement to apply for a U.S. student visa. The
author then concluded that since Peter meets the requirement, he will necessarily get the visa. It is
obviously that this conclusion can not be properly drawn because application can be denied even
that the applicant meets the basic requirement. Therefore, A is the best answer.

It seems easy. Now take a high-level question.

Example #2

Something must be done to stop spam. In early days, people seldom received unsolicited
email advertisement; but now that numerous bulk email software and email address finders
are developed to collect email address all around the world. Advertisers use email addresses
to market their products and even sell such email lists to other advertisers. As a result, almost
everyone ever get junk email, and sometime several and even tens of annoying emails a day.
So, relevant anti-spam regulations should be framed to stop unsolicited advertising.

The two portions in boldface play which of the following roles?

113
A. Background that the argument depends on and conclusion that can be drawn from the
argument.

B. Part of evidence that the argument includes, and inference that can be drawn from this
passage.

C. Pre-evidence that the argument depends on and part of evidence that supports the conclusion.

D. Background that argument depends on and part of evidence that supports the conclusion.

E. Pre-evidence that argument includes and a method that helps to supports that conclusion.

The first portion in boldface introduced a previous situation, as compared to current situation. The
author then made the conclusion in the last sentence, or the second portion in boldface. Therefore,
choice B is the best answer.

The End of GMAT Verbal Study Guide

114

También podría gustarte